AH2 Exam 1

अब Quizwiz के साथ अपने होमवर्क और परीक्षाओं को एस करें!

When planning emergent care for a patient with a suspected MI, what should the nurse anticipate administrating? A Oxygen, nitroglycerin, aspirin, and morphine B Oxygen, furosemide (Lasix), nitroglycerin, and meperidine C Aspirin, nitroprusside (Nipride), dopamine (Intropin), and oxygen D Nitroglycerin, lorazepam (Ativan), oxygen, and warfarin (Coumadin)

A Oxygen, nitroglycerin, aspirin, and morphine The American Heart Association's guidelines for emergency care of the patient with chest pain include the administration of oxygen, nitroglycerin, aspirin, and morphine. These interventions serve to relieve chest pain, improve oxygenation, decrease myocardial workload, and prevent further platelet aggregation. The other medications may be used later in the patient's treatment.

When providing nutritional counseling for patients at risk for CAD, which foods would the nurse encourage patients to include in their diet (select all that apply)? A Tofu B Walnuts C Tuna fish D Whole milk E Orange juice

A Tofu B Walnuts C Tuna fish Tuna fish, tofu, and walnuts are all rich in omega-3 fatty acids, which have been shown to reduce the risks associated with CAD when consumed regularly.

The community health nurse is planning health promotion teaching targeted at preventing coronary artery disease (CAD). Which ethnic group would the nurse select as the highest priority for this intervention? A White male B Hispanic male C African American male D Native American female

A White male The incidence of CAD and myocardial infarction (MI) is highest among white, middle-aged men. Hispanic individuals have lower rates of CAD than non-Hispanic whites or African Americans. African Americans have an earlier age of onset and more severe CAD than whites and more than twice the mortality rate of whites of the same age. Native Americans have increased mortality in less than 35-year-olds and have major modifiable risk factors such as diabetes.

A 52-year-old male patient has received a bolus dose and an infusion of alteplase (Activase) for an ST-segment elevation myocardial infarction (STEMI). To determine the effectiveness of this medication, the nurse should assess the patient for the A presence of chest pain. B blood in the urine or stool. C tachycardia with hypotension. D decreased level of consciousness.

A presence of chest pain. Alteplase is a fibrinolytic that is administered to patients who have had an STEMI. If the medication is effective, the patient's chest pain will resolve because the medication dissolves the thrombus in the coronary artery and results in reperfusion of the myocardium. Bleeding is a major complication of fibrinolytic therapy. Signs of major bleeding include decreased level of consciousness, blood in the urine or stool, and increased heart rate with decreased blood pressure.

In evaluating an asthmatic patient's knowledge of self care, the nurse recognizes that additional instruction is needed when the patient says? A. "I use my corticosteroid inhaler when I feel short of breath" B. "I get a flu shot every year and see my health care provider if I have an upper respiratory infection" C. "I use my inhaler before I visit my aunt who has a cat, but I only visit for a few minutes because of my allergies" D. "I walk for 30 minutes every day but sometimes I have to use my bronchodilator inhaler before walking to prevent me from getting short of breath"

A. "I use my corticosteroid inhaler when I feel short of breath"

When admitting a patient with a diagnosis of asthma exacerbation, the nurse will assess for what potential triggers? Select all that apply. A. Exercise B. Allergies C. Emotional stress D. Decreased humidity E. Upper respiratory infections

A. Exercise B. Allergies C. Emotional stress E. Upper respiratory infections Although the exact mechanism of asthma is unknown, there are several triggers that may precipitate an attack. These include allergens, exercise, air pollutants, upper respiratory infections, drug and food additives, psychologic factors, and gastroesophageal reflux disease (GERD).

A plan of care for the patient with COPD could include? Select all that apply. A. Exercise such as walking B. High flow rate of O2 administration C. Low-dose chronic oral corticosteroid therapy D. Use of peak flow meter to monitor the progression of COPD E. Breathing exercises such as pursed lip breathing that focus on exhalation

A. Exercise such as walking E. Breathing exercises such as pursed lip breathing that focus on exhalation

Which statements by a patient with moderate asthma inform the nurse that the patient needs more teaching about medications? Select all that apply. A. If I can't afford all of my medicines, I will only use the salmeterol B. I will stay inside if there is a high pollen count to prevent having an asthma attack C. I will rinse my mouth out after using fluticasone to prevent oral candidiasis D. I must have omalizumab injected every 2 to 4 weeks because inhalers don't help my asthma E. I can use my inhaler three times, every 20 minutes, before going to the hospital if my peak flow has not improved F. My GERD medications will help my asthma and my asthma medications will help my GERD

A. If I can't afford all of my medicines, I will only use the salmeterol F. My GERD medications will help my asthma and my asthma medications will help my GERD

While teaching a patient with asthma about the appropriate use of a peak flow meter, what should the nurse instruct the patient to do? A. Keep a record of the peak flow meter numbers if symptoms of asthma are getting worse B. Use the flow meter each morning after taking medications to evaluate their effectiveness C. Increase the doses of the long-term control medication if the peak flow numbers decrease D. Empty the lungs and then inhale quickly through the mouthpiece to measure how fast air can be inhaled

A. Keep a record of the peak flow meter numbers if symptoms of asthma are getting worse

The nurse notes new onset confusion in an older patient who is normally alert and oriented. In which order should the nurse take the following actions? (Put a comma and a space between each answer choice [A, B, C, D].) a. Obtain the oxygen saturation. b. Check the patient's pulse rate. c. Document the change in status. d. Notify the health care provider

ANS: A, B, D, C Assessment for physiologic causes of new onset confusion such as pneumonia, infection, or perfusion problems should be the first action by the nurse. Airway and oxygenation should be assessed first, then circulation. After assessing the patient, the nurse should notify the health care provider. Finally, documentation of the assessments and care should be done

The nurse assumes care of a patient who just returned from surgery for a total laryngectomy and radical neck dissection and notes the following problems. In which order should the nurse address the problems? (Put a comma and a space between each answer choice [A, B, C, D].) a. The patient is in a side-lying position with the head of the bed flat. b. The patient is coughing blood-tinged secretions from the tracheostomy. c. The nasogastric (NG) tube is disconnected from suction and clamped off. d. The wound drain in the neck incision contains 200 mL of bloody drainage

ANS: A, B, D, C The patient should first be placed in a semi-Fowler's position to maintain the airway and reduce incisional swelling. The blood-tinged secretions may obstruct the airway, so suctioning is the next appropriate action. Then the wound drain should be drained because the 200 mL of drainage will decrease the amount of suction in the wound drain and could lead to incisional swelling and poor healing. Finally, the NG tube should be reconnected to suction to prevent gastric dilation, nausea, and vomiting

When preparing to defibrillate a patient. In which order will the nurse perform the following steps? (Put a comma and a space between each answer choice [A, B, C, D, E].) a. Turn the defibrillator on. b. Deliver the electrical charge. c. Select the appropriate energy level. d. Place the paddles on the patient's chest. e. Check the location of other staff and call out "all clear."

ANS: A, C, D, E, B This order will result in rapid defibrillation without endangering hospital staff.

Which action will the nurse include in the plan of care for a patient who was admitted with syncopal episodes of unknown origin? a. Instruct the patient to call for assistance before getting out of bed. b. Explain the association between various dysrhythmias and syncope. c. Educate the patient about the need to avoid caffeine and other stimulants. d. Tell the patient about the benefits of implantable cardioverter-defibrillators.

ANS: A A patient with fainting episodes is at risk for falls. The nurse will plan to minimize the risk by having assistance whenever the patient up. The other actions may be needed if dysrhythmias are found to be the cause of the patient's syncope, but are not appropriate for syncope of unknown origin

Which action by a new registered nurse (RN) who is orienting to the progressive care unit indicates a good understanding of the treatment of cardiac dysrhythmias? a. Injects IV adenosine (Adenocard) over 2 seconds to a patient with supraventricular tachycardia b. Obtains the defibrillator and quickly brings it to the bedside of a patient whose monitor shows asystole c. Turns the synchronizer switch to the "on" position before defibrillating a patient with ventricular fibrillation d. Gives the prescribed dose of diltiazem (Cardizem) to a patient with new-onset type II second degree AV block

ANS: A Adenosine must be given over 1 to 2 seconds to be effective. The other actions indicate a need for more education about treatment of cardiac dysrhythmias. The RN should hold the diltiazem until talking to the health care provider. The treatment for asystole is immediate CPR. The synchronizer switch should be "off" when defibrillating

The nurse has just finished teaching a hypertensive patient about the newly prescribed ramipril (Altace). Which patient statement indicates that more teaching is needed? a. "A little swelling around my lips and face is okay." b. "The medication may not work as well if I take any aspirin." c. "The doctor may order a blood potassium level occasionally." d. "I will call the doctor if I notice that I have a frequent cough."

ANS: A Angioedema occurring with angiotensin-converting enzyme (ACE) inhibitor therapy is an indication that the ACE inhibitor should be discontinued. The patient should be taught that if any swelling of the face or oral mucosa occurs, the health care provider should be immediately notified because this could be life threatening. The other patient statements indicate that the patient has an accurate understanding of ACE inhibitor therapy

A patient with dilated cardiomyopathy has new onset atrial fibrillation that has been unresponsive to drug therapy for several days. The priority teaching needed for this patient would include information about a. anticoagulant therapy. b. permanent pacemakers. c. electrical cardioversion. d. IV adenosine (Adenocard).

ANS: A Atrial fibrillation therapy that has persisted for more than 48 hours requires anticoagulant treatment for 3 weeks before attempting cardioversion. This is done to prevent embolization of clots from the atria. Cardioversion may be done after several weeks of anticoagulation therapy. Adenosine is not used to treat atrial fibrillation. Pacemakers are routinely used for patients with bradydysrhythmias. Information does not indicate that the patient has a slow heart rate

After the nurse gives IV atropine to a patient with symptomatic type 1, second-degree atrioventricular (AV) block, which finding indicates that the medication has been effective? a. Increase in the patient's heart rate b. Increase in strength of peripheral pulses c. Decrease in premature atrial contractions d. Decrease in premature ventricular contractions

ANS: A Atropine will increase the heart rate and conduction through the AV node. Because the medication increases electrical conduction, not cardiac contractility, the quality of the peripheral pulses is not used to evaluate the drug effectiveness. The patient does not have premature atrial or ventricular contractions

A patient with acute dyspnea is scheduled for a spiral computed tomography (CT) scan. Which information obtained by the nurse is a priority to communicate to the health care provider before the CT? a. Allergy to shellfish b. Apical pulse of 104 c. Respiratory rate of 30 d. Oxygen saturation of 90%

ANS: A Because iodine-based contrast media is used during a spiral CT, the patient may need to have the CT scan without contrast or be premedicated before injection of the contrast media. The increased pulse, low oxygen saturation, and tachypnea all indicate a need for further assessment or intervention but do not indicate a need to modify the CT procedure

The clinic nurse teaches a patient with a 42 pack-year history of cigarette smoking about lung disease. Which information will be most important for the nurse to include? a. Options for smoking cessation b. Reasons for annual sputum cytology testing c. Erlotinib (Tarceva) therapy to prevent tumor risk d. Computed tomography (CT) screening for lung cancer

ANS: A Because smoking is the major cause of lung cancer, the most important role for the nurse is teaching patients about the benefits of and means of smoking cessation. CT scanning is currently being investigated as a screening test for high-risk patients. However, if there is a positive finding, the person already has lung cancer. Erlotinib may be used in patients who have lung cancer, but it is not used to reduce the risk of developing cancer

The nurse is caring for a 78-year-old patient with aortic stenosis. Which assessment data obtained by the nurse would be most important to report to the health care provider? a. The patient complains of chest pressure when ambulating. b. A loud systolic murmur is heard along the right sternal border. c. A thrill is palpated at the second intercostal space, right sternal border. d. The point of maximum impulse (PMI) is at the left midclavicular line.

ANS: A Chest pressure (or pain) occurring with aortic stenosis is caused by cardiac ischemia, and reporting this information would be a priority. A systolic murmur and thrill are expected in a patient with aortic stenosis. A PMI at the left midclavicular line is normal

After being hit by a baseball, a patient arrives in the emergency department with a possible nasal fracture. Which finding by the nurse is most important to report to the health care provider? a. Clear nasal drainage b. Complaint of nasal pain c. Bilateral nose swelling and bruising d. Inability to breathe through the nose

ANS: A Clear nasal drainage may indicate a meningeal tear with leakage of cerebrospinal fluid. This would place the patient at risk for complications such as meningitis. The other findings are typical with a nasal fracture and do not indicate any complications

The nurse administers prescribed therapies for a patient with cor pulmonale and right-sided heart failure. Which assessment would best evaluate the effectiveness of the therapies? a. Observe for distended neck veins. b. Auscultate for crackles in the lungs. c. Palpate for heaves or thrills over the heart. d. Review hemoglobin and hematocrit values.

ANS: A Cor pulmonale is right ventricular failure caused by pulmonary hypertension, so clinical manifestations of right ventricular failure such as peripheral edema, jugular venous distention, and right upper-quadrant abdominal tenderness would be expected. Crackles in the lungs are likely to be heard with left-sided heart failure. Findings in cor pulmonale include evidence of right ventricular hypertrophy on electrocardiogram ECG and an increase in intensity of the second heart sound. Heaves or thrills are not common with cor pulmonale. Chronic hypoxemia leads to polycythemia and increased total blood volume and viscosity of the blood. The hemoglobin and hematocrit values are more likely to be elevated with cor pulmonale than decreased

A patient with bacterial pneumonia has rhonchi and thick sputum. What is the nurse's most appropriate action to promote airway clearance? a. Assist the patient to splint the chest when coughing. b. Teach the patient about the need for fluid restrictions. c. Encourage the patient to wear the nasal oxygen cannula. d. Instruct the patient on the pursed lip breathing technique.

ANS: A Coughing is less painful and more likely to be effective when the patient splints the chest during coughing. Fluids should be encouraged to help liquefy secretions. Nasal oxygen will improve gas exchange, but will not improve airway clearance. Pursed lip breathing is used to improve gas exchange in patients with COPD, but will not improve airway clearance

On auscultation of a patient's lungs, the nurse hears low-pitched, bubbling sounds during inhalation in the lower third of both lungs. How should the nurse document this finding? a. Inspiratory crackles at the bases b. Expiratory wheezes in both lungs c. Abnormal lung sounds in the apices of both lungs d. Pleural friction rub in the right and left lower lobes

ANS: A Crackles are low-pitched, bubbling sounds usually heard on inspiration. Wheezes are high-pitched sounds. They can be heard during the expiratory or inspiratory phase of the respiratory cycle. The lower third of both lungs are the bases, not apices. Pleural friction rubs are grating sounds that are usually heard during both inspiration and expiration

The nurse is caring for a 64-year-old patient admitted with mitral valve regurgitation. Which information obtained by the nurse when assessing the patient should be communicated to the health care provider immediately? a. The patient has bilateral crackles. b. The patient has bilateral, 4+ peripheral edema. c. The patient has a loud systolic murmur across the precordium. d. The patient has a palpable thrill felt over the left anterior chest.

ANS: A Crackles that are audible throughout the lungs indicate that the patient is experiencing severe left ventricular failure with pulmonary congestion and needs immediate interventions such as diuretics. A systolic murmur and palpable thrill would be expected in a patient with mitral regurgitation. Although 4+ peripheral edema indicates a need for a change in therapy, it does not need to be addressed urgently

To auscultate for S3 or S4 gallops in the mitral area, the nurse listens with the a. bell of the stethoscope with the patient in the left lateral position. b. diaphragm of the stethoscope with the patient in a supine position. c. bell of the stethoscope with the patient sitting and leaning forward. d. diaphragm of the stethoscope with the patient lying flat on the left side.

ANS: A Gallop rhythms generate low-pitched sounds and are most easily heard with the bell of the stethoscope. Sounds associated with the mitral valve are accentuated by turning the patient to the left side, which brings the heart closer to the chest wall. The diaphragm of the stethoscope is best to use for the higher-pitched sounds such as S1 and S2

A 20-year-old has a mandatory electrocardiogram (ECG) before participating on a college soccer team and is found to have sinus bradycardia, rate 52. Blood pressure (BP) is 114/54, and the student denies any health problems. What action by the nurse is most appropriate? a. Allow the student to participate on the soccer team. b. Refer the student to a cardiologist for further diagnostic testing. c. Tell the student to stop playing immediately if any dyspnea occurs. d. Obtain more detailed information about the student's family health history.

ANS: A In an aerobically trained individual, sinus bradycardia is normal. The student's normal BP and negative health history indicate that there is no need for a cardiology referral or for more detailed information about the family's health history. Dyspnea during an aerobic activity such as soccer is normal

The nurse assesses the chest of a patient with pneumococcal pneumonia. Which finding would the nurse expect? a. Increased tactile fremitus b. Dry, nonproductive cough c. Hyperresonance to percussion d. A grating sound on auscultation

ANS: A Increased tactile fremitus over the area of pulmonary consolidation is expected with bacterial pneumonias. Dullness to percussion would be expected. Pneumococcal pneumonia typically presents with a loose, productive cough. Adventitious breath sounds such as crackles and wheezes are typical. A grating sound is more representative of a pleural friction rub rather than pneumonia

The nurse is caring for a mechanically ventilated patient with a cuffed tracheostomy tube. Which action by the nurse would best determine if the cuff has been properly inflated? a. Use a manometer to ensure cuff pressure is at an appropriate level. b. Check the amount of cuff pressure ordered by the health care provider. c. Suction the patient first with a fenestrated inner cannula to clear secretions. d. Insert the decannulation plug before the nonfenestrated inner cannula is removed.

ANS: A Measurement of cuff pressure using a manometer to ensure that cuff pressure is 20 mm Hg or lower will avoid compression of the tracheal wall and capillaries. Never insert the decannulation plug in a tracheostomy tube until the cuff is deflated and the nonfenestrated inner cannula is removed. Otherwise, the patient's airway is occluded. A health care provider's order is not required to determine safe cuff pressure. A nonfenestrated inner cannula must be used to suction a patient to prevent tracheal damage occurring from the suction catheter passing through the fenestrated openings

A patient in the intensive care unit with acute decompensated heart failure (ADHF) complains of severe dyspnea and is anxious, tachypneic, and tachycardic. All of the following medications have been ordered for the patient. The nurse's priority action will be to a. give IV morphine sulfate 4 mg. b. give IV diazepam (Valium) 2.5 mg. c. increase nitroglycerin (Tridil) infusion by 5 mcg/min. d. increase dopamine (Intropin) infusion by 2 mcg/kg/min.

ANS: A Morphine improves alveolar gas exchange, improves cardiac output by reducing ventricular preload and afterload, decreases anxiety, and assists in reducing the subjective feeling of dyspnea. Diazepam may decrease patient anxiety, but it will not improve the cardiac output or gas exchange. Increasing the dopamine may improve cardiac output, but it will also increase the heart rate and myocardial oxygen consumption. Nitroglycerin will improve cardiac output and may be appropriate for this patient, but it will not directly reduce anxiety and will not act as quickly as morphine to decrease dyspnea

Which action should the nurse include in the plan of care when caring for a patient admitted with acute decompensated heart failure (ADHF) who is receiving nesiritide (Natrecor)? a. Monitor blood pressure frequently. b. Encourage patient to ambulate in room. c. Titrate nesiritide slowly before stopping. d. Teach patient about home use of the drug.

ANS: A Nesiritide is a potent arterial and venous dilator, and the major adverse effect is hypotension. Because the patient is likely to have orthostatic hypotension, the patient should not be encouraged to ambulate. Nesiritide does not require titration and is used for ADHF but not in a home setting

An older patient is receiving standard multidrug therapy for tuberculosis (TB). The nurse should notify the health care provider if the patient exhibits which finding? a. Yellow-tinged skin b. Orange-colored sputum c. Thickening of the fingernails d. Difficulty hearing high-pitched voices

ANS: A Noninfectious hepatitis is a toxic effect of isoniazid (INH), rifampin, and pyrazinamide, and patients who develop hepatotoxicity will need to use other medications. Changes in hearing and nail thickening are not expected with the four medications used for initial TB drug therapy. Presbycusis is an expected finding in the older adult patient. Orange discoloration of body fluids is an expected side effect of rifampin and not an indication to call the health care provider

Propranolol (Inderal) is prescribed for a patient diagnosed with hypertension. The nurse should consult with the health care provider before giving this medication when the patient reveals a history of a. asthma. b. daily alcohol use. c. peptic ulcer disease. d. myocardial infarction (MI).

ANS: A Nonselective b-blockers block b1- and b2-adrenergic receptors and can cause bronchospasm, especially in patients with a history of asthma. b-Blockers will have no effect on the patient's peptic ulcer disease or alcohol use. b-Blocker therapy is recommended after MI

The nurse is caring for a hospitalized older patient who has nasal packing in place to treat a nosebleed. Which assessment finding will require the most immediate action by the nurse? a. The oxygen saturation is 89%. b. The nose appears red and swollen. c. The patient's temperature is 100.1° F (37.8° C). d. The patient complains of level 8 (0 to 10 scale) pain.

ANS: A Older patients with nasal packing are at risk of aspiration or airway obstruction. An O2 saturation of 89% should alert the nurse to further assess for these complications. The other assessment data also indicate a need for nursing action but not as immediately as the low O2 saturation

The nurse cares for a patient who has just had a thoracentesis. Which assessment information obtained by the nurse is a priority to communicate to the health care provider? a. Oxygen saturation is 88%. b. Blood pressure is 145/90 mm Hg. c. Respiratory rate is 22 breaths/minute when lying flat. d. Pain level is 5 (on 0 to 10 scale) with a deep breath.

ANS: A Oxygen saturation would be expected to improve after a thoracentesis. A saturation of 88% indicates that a complication such as pneumothorax may be occurring. The other assessment data also indicate a need for ongoing assessment or intervention, but the low oxygen saturation is the priority

A patient experiences a chest wall contusion as a result of being struck in the chest with a baseball bat. The emergency department nurse would be most concerned if which finding is observed during the initial assessment? a. Paradoxic chest movement b. Complaint of chest wall pain c. Heart rate of 110 beats/minute d. Large bruised area on the chest

ANS: A Paradoxic chest movement indicates that the patient may have flail chest, which can severely compromise gas exchange and can rapidly lead to hypoxemia. Chest wall pain, a slightly elevated pulse rate, and chest bruising all require further assessment or intervention, but the priority concern is poor gas exchange

A nurse obtains a health history from a patient who has a 35 pack-year smoking history. The patient complains of hoarseness and tightness in the throat and difficulty swallowing. Which question is most important for the nurse to ask? a. "How much alcohol do you drink in an average week?" b. "Do you have a family history of head or neck cancer?" c. "Have you had frequent streptococcal throat infections?" d. "Do you use antihistamines for upper airway congestion?"

ANS: A Prolonged alcohol use and smoking are associated with the development of laryngeal cancer, which the patient's symptoms and history suggest. Family history is not a risk factor for head or neck cancer. Frequent antihistamine use would be asked about if the nurse suspected allergic rhinitis, but the patient's symptoms are not suggestive of this diagnosis. Streptococcal throat infections also may cause these clinical manifestations, but patients with this type of infection will also have pain and a fever

The nurse suspects cardiac tamponade in a patient who has acute pericarditis. To assess for the presence of pulsus paradoxus, the nurse should a. note when Korotkoff sounds are auscultated during both inspiration and expiration. b. subtract the diastolic blood pressure (DBP) from the systolic blood pressure (SBP). c. check the electrocardiogram (ECG) for variations in rate during the respiratory cycle. d. listen for a pericardial friction rub that persists when the patient is instructed to stop breathing.

ANS: A Pulsus paradoxus exists when there is a gap of greater than 10 mm Hg between when Korotkoff sounds can be heard during only expiration and when they can be heard throughout the respiratory cycle. The other methods described would not be useful in determining the presence of pulsus paradoxus

While caring for a patient with aortic stenosis, the nurse identifies a nursing diagnosis of acute pain related to decreased coronary blood flow. A priority nursing intervention for this patient would be to a. promote rest to decrease myocardial oxygen demand. b. teach the patient about the need for anticoagulant therapy. c. teach the patient to use sublingual nitroglycerin for chest pain. d. raise the head of the bed 60 degrees to decrease venous return.

ANS: A Rest is recommended to balance myocardial oxygen supply and demand and to decrease chest pain. The patient with aortic stenosis requires higher preload to maintain cardiac output, so nitroglycerin and measures to decrease venous return are contraindicated. Anticoagulation is not recommended unless the patient has atrial fibrillation

A patient is admitted to the emergency department complaining of sudden onset shortness of breath and is diagnosed with a possible pulmonary embolus. How should the nurse prepare the patient for diagnostic testing to confirm the diagnosis? a. Start an IV so contrast media may be given. b. Ensure that the patient has been NPO for at least 6 hours. c. Inform radiology that radioactive glucose preparation is needed. d. Instruct the patient to undress to the waist and remove any metal objects.

ANS: A Spiral computed tomography (CT) scans are the most commonly used test to diagnose pulmonary emboli, and contrast media may be given IV. A chest x-ray may be ordered but will not be diagnostic for a pulmonary embolus. Preparation for a chest x-ray includes undressing and removing any metal. Bronchoscopy is used to detect changes in the bronchial tree, not to assess for vascular changes, and the patient should be NPO 6 to 12 hours before the procedure. Positron emission tomography (PET) scans are most useful in determining the presence of malignancy, and a radioactive glucose preparation is used

The charge nurse observes a new registered nurse (RN) doing discharge teaching for a patient with hypertension who has a new prescription for enalapril (Vasotec). The charge nurse will need to intervene if the new RN tells the patient to a. increase the dietary intake of high-potassium foods. b. make an appointment with the dietitian for teaching. c. check the blood pressure (BP) with a home BP monitor at least once a day. d. move slowly when moving from lying to sitting to standing.

ANS: A The ACE inhibitors cause retention of potassium by the kidney, so hyperkalemia is a possible adverse effect. The other teaching by the new RN is appropriate for a patient with newly diagnosed hypertension who has just started therapy with enalapril

The nurse hears a murmur between the S1 and S2 heart sounds at the patient's left fifth intercostal space and midclavicular line. How will the nurse record this information? a. Systolic murmur heard at mitral area b. Systolic murmur heard at Erb's point c. Diastolic murmur heard at aortic area d. Diastolic murmur heard at the point of maximal impulse

ANS: A The S1 signifies the onset of ventricular systole. S2 signifies the onset of diastole. A murmur occurring between these two sounds is a systolic murmur. The mitral area is the intersection of the left fifth intercostal space and the midclavicular line. The other responses describe murmurs heard at different landmarks on the chest and/or during the diastolic phase of the cardiac cycle

Which action should the nurse take first when a patient develops a nosebleed? a. Pinch the lower portion of the nose for 10 minutes. b. Pack the affected nare tightly with an epistaxis balloon. c. Obtain silver nitrate that will be needed for cauterization. d. Apply ice compresses over the patient's nose and cheeks.

ANS: A The first nursing action for epistaxis is to apply direct pressure by pinching the nostrils. Application of cold packs may decrease blood flow to the area, but will not be sufficient to stop bleeding. Cauterization and nasal packing are medical interventions that may be needed if pressure to the nares does not stop the bleeding, but these are not the first actions to take for a nosebleed

The nurse assesses a patient with chronic obstructive pulmonary disease (COPD) who has been admitted with increasing dyspnea over the last 3 days. Which finding is most important for the nurse to report to the health care provider? a. Respirations are 36 breaths/minute. b. Anterior-posterior chest ratio is 1:1. c. Lung expansion is decreased bilaterally. d. Hyperresonance to percussion is present.

ANS: A The increase in respiratory rate indicates respiratory distress and a need for rapid interventions such as administration of oxygen or medications. The other findings are common chronic changes occurring in patients with COPD

Which nursing action should the nurse take first in order to assist a patient with newly diagnosed stage 1 hypertension in making needed dietary changes? a. Collect a detailed diet history. b. Provide a list of low-sodium foods. c. Help the patient make an appointment with a dietitian. d. Teach the patient about foods that are high in potassium.

ANS: A The initial nursing action should be assessment of the patient's baseline dietary intake through a thorough diet history. The other actions may be appropriate, but assessment of the patient's baseline should occur first

The standard policy on the cardiac unit states, "Notify the health care provider for mean arterial pressure (MAP) less than 70 mm Hg." The nurse will need to call the health care provider about the a. postoperative patient with a BP of 116/42. b. newly admitted patient with a BP of 150/87. c. patient with left ventricular failure who has a BP of 110/70. d. patient with a myocardial infarction who has a BP of 140/86.

ANS: A The mean arterial pressure (MAP) is calculated using the formula MAP = (systolic BP + 2 diastolic BP)/3. The MAP for the postoperative patient in answer 3 is 67. The MAP in the other three patients is higher than 70 mm Hg

Which intervention by a new nurse who is caring for a patient who has just had an implantable cardioverter-defibrillator (ICD) inserted indicates a need for more education about care of patients with ICDs? a. The nurse assists the patient to do active range of motion exercises for all extremities. b. The nurse assists the patient to fill out the application for obtaining a Medic Alert ID. c. The nurse gives amiodarone (Cordarone) to the patient without first consulting with the health care provider. d. The nurse teaches the patient that sexual activity usually can be resumed once the surgical incision is healed.

ANS: A The patient should avoid moving the arm on the ICD insertion site until healing has occurred in order to prevent displacement of the ICD leads. The other actions by the new nurse are appropriate for this patient.

Which patient in the ear, nose, and throat (ENT) clinic should the nurse assess first? a. A 23-year-old who is complaining of a sore throat and has a muffled voice b. A 34-year-old who has a "scratchy throat" and a positive rapid strep antigen test c. A 55-year-old who is receiving radiation for throat cancer and has severe fatigue d. A 72-year-old with a history of a total laryngectomy whose stoma is red and inflamed

ANS: A The patient's clinical manifestation of a muffled voice suggests a possible peritonsillar abscess that could lead to an airway obstruction requiring rapid assessment and potential treatment. The other patients do not have diagnoses or symptoms that indicate any life-threatening problems

Two days after an acute myocardial infarction (MI), a patient complains of stabbing chest pain that increases with a deep breath. Which action will the nurse take first? a. Auscultate the heart sounds. b. Check the patient's temperature. c. Notify the patient's health care provider. d. Give the PRN acetaminophen (Tylenol).

ANS: A The patient's clinical manifestations and history are consistent with pericarditis, and the first action by the nurse should be to listen for a pericardial friction rub. Checking the temperature and notifying the health care provider are also appropriate actions but would not be done before listening for a rub. It is not stated for what symptom (e.g., headache) or finding (e.g., increased temperature) the PRN acetaminophen (Tylenol) is ordered.

A patient's cardiac monitor shows a pattern of undulations of varying contours and amplitude with no measurable ECG pattern. The patient is unconscious and pulseless. Which action should the nurse take first? a. Perform immediate defibrillation. b. Give epinephrine (Adrenalin) IV. c. Prepare for endotracheal intubation. d. Give ventilations with a bag-valve-mask device.

ANS: A The patient's rhythm and assessment indicate ventricular fibrillation and cardiac arrest; the initial action should be to defibrillate. If a defibrillator is not immediately available or is unsuccessful in converting the patient to a better rhythm, the other actions may be appropriate

A patient who was admitted the previous day with pneumonia complains of a sharp pain of 7 (based on 0 to 10 scale) "whenever I take a deep breath." Which action will the nurse take next? a. Auscultate breath sounds. b. Administer the PRN morphine. c. Have the patient cough forcefully. d. Notify the patient's health care provider.

ANS: A The patient's statement indicates that pleurisy or a pleural effusion may have developed and the nurse will need to listen for a pleural friction rub and/or decreased breath sounds. Assessment should occur before administration of pain medications. The patient is unlikely to be able to cough forcefully until pain medication has been administered. The nurse will want to obtain more assessment data before calling the health care provider

During a visit to a 78-year-old with chronic heart failure, the home care nurse finds that the patient has ankle edema, a 2-kg weight gain over the past 2 days, and complains of "feeling too tired to get out of bed." Based on these data, the best nursing diagnosis for the patient is a. activity intolerance related to fatigue. b. disturbed body image related to weight gain. c. impaired skin integrity related to ankle edema. d. impaired gas exchange related to dyspnea on exertion.

ANS: A The patient's statement supports the diagnosis of activity intolerance. There are no data to support the other diagnoses, although the nurse will need to assess for other patient problems

The nurse completes discharge instructions for a patient with a total laryngectomy. Which statement by the patient indicates that additional instruction is needed? a. "I must keep the stoma covered with an occlusive dressing at all times." b. "I can participate in most of my prior fitness activities except swimming." c. "I should wear a Medic-Alert bracelet that identifies me as a neck breather." d. "I need to be sure that I have smoke and carbon monoxide detectors installed."

ANS: A The stoma may be covered with clothing or a loose dressing, but this is not essential. An occlusive dressing will completely block the patient's airway. The other patient comments are all accurate and indicate that the teaching has been effective

The nurse is caring for a patient who has a right-sided chest tube after a right lower lobectomy. Which nursing action can the nurse delegate to the unlicensed assistive personnel (UAP)? a. Document the amount of drainage every eight hours. b. Obtain samples of drainage for culture from the system. c. Assess patient pain level associated with the chest tube. d. Check the water-seal chamber for the correct fluid level.

ANS: A UAP education includes documentation of intake and output. The other actions are within the scope of practice and education of licensed nursing personnel

Which statement by the patient indicates that the teaching has been effective for a patient scheduled for radiation therapy of the larynx? a. "I will need to buy a water bottle to carry with me." b. "I should not use any lotions on my neck and throat." c. "Until the radiation is complete, I may have diarrhea." d. "Alcohol-based mouthwashes will help clean oral ulcers."

ANS: A Xerostomia can be partially alleviated by drinking fluids at frequent intervals. Radiation will damage tissues at the site being radiated but should not affect the abdominal organs, so loose stools are not a usual complication of head and neck radiation therapy. Frequent oral rinsing with non-alcohol-based rinses is recommended. Prescribed lotions and sunscreen may be used on radiated skin, although they should not be used just before the radiation therapy

The nurse is reviewing the medical records for five patients who are scheduled for their yearly physical examinations in September. Which patients should receive the inactivated influenza vaccination (select all that apply)? a. A 76-year-old nursing home resident b. A 36-year-old female patient who is pregnant c. A 42-year-old patient who has a 15 pack-year smoking history d. A 30-year-old patient who takes corticosteroids for rheumatoid arthritis e. A 24-year-old patient who has allergies to penicillin and cephalosporins

ANS: A, B, D Current guidelines suggest that healthy individuals between 6 months and age 49 receive intranasal immunization with live, attenuated influenza vaccine. Individuals who are pregnant, residents of nursing homes, or are immunocompromised or who have chronic medical conditions should receive inactivated vaccine by injection. The corticosteroid use by the 30-year-old increases the risk for infection

Based on the Joint Commission Core Measures for patients with heart failure, which topics should the nurse include in the discharge teaching plan for a patient who has been hospitalized with chronic heart failure (select all that apply)? a. How to take and record daily weight b. Importance of limiting aerobic exercise c. Date and time of follow-up appointment d. Symptoms indicating worsening heart failure e. Actions and side effects of prescribed medications

ANS: A, C, D, E The Joint Commission Core Measures state that patients should be taught about prescribed medications, follow-up appointments, weight monitoring, and actions to take for worsening symptoms. Patients with heart failure are encouraged to begin or continue aerobic exercises such as walking, while self-monitoring to avoid excessive fatigue

The clinic nurse is teaching a patient with acute sinusitis. Which interventions should the nurse plan to include in the teaching session (select all that apply)? a. Decongestants can be used to relieve swelling. b. Blowing the nose should be avoided to decrease the nosebleed risk. c. Taking a hot shower will increase sinus drainage and decrease pain. d. Saline nasal spray can be made at home and used to wash out secretions. e. You will be more comfortable if you keep your head in an upright position.

ANS: A, C, D, E The steam and heat from a shower will help thin secretions and improve drainage. Decongestants can be used to relieve swelling. Patients can use either over-the-counter (OTC) sterile saline solutions or home-prepared saline solutions to thin and remove secretions. Maintaining an upright posture decreases sinus pressure and the resulting pain. Blowing the nose after a hot shower or using the saline spray is recommended to expel secretions.

When auscultating over the patient's abdominal aorta, the nurse hears a humming sound. The nurse documents this finding as a a. thrill. b. bruit. c. murmur. d. normal finding.

ANS: B A bruit is the sound created by turbulent blood flow in an artery. Thrills are palpable vibrations felt when there is turbulent blood flow through the heart or in a blood vessel. A murmur is the sound caused by turbulent blood flow through the heart. Auscultating a bruit in an artery is not normal and indicates pathology

When caring for a patient who is hospitalized with active tuberculosis (TB), the nurse observes a student nurse who is assigned to take care of a patient. Which action, if performed by the student nurse, would require an intervention by the nurse? a. The patient is offered a tissue from the box at the bedside. b. A surgical face mask is applied before visiting the patient. c. A snack is brought to the patient from the unit refrigerator. d. Hand washing is performed before entering the patient's room.

ANS: B A high-efficiency particulate-absorbing (HEPA) mask, rather than a standard surgical mask, should be used when entering the patient's room because the HEPA mask can filter out 100% of small airborne particles. Hand washing before entering the patient's room is appropriate. Because anorexia and weight loss are frequent problems in patients with TB, bringing food to the patient is appropriate. The student nurse should perform hand washing after handling a tissue that the patient has used, but no precautions are necessary when giving the patient an unused tissue

A patient is scheduled for a cardiac catheterization with coronary angiography. Before the test, the nurse informs the patient that a. it will be important to lie completely still during the procedure. b. a flushed feeling may be noted when the contrast dye is injected. c. monitored anesthesia care will be provided during the procedure. d. arterial pressure monitoring will be required for 24 hours after the test.

ANS: B A sensation of warmth or flushing is common when the contrast material is injected, which can be anxiety-producing unless it has been discussed with the patient. The patient may receive a sedative drug before the procedure, but monitored anesthesia care is not used. Arterial pressure monitoring is not routinely used after the procedure to monitor blood pressure. The patient is not immobile during cardiac catheterization and may be asked to cough or take deep breaths

A patient with idiopathic pulmonary arterial hypertension (IPAH) is receiving nifedipine (Procardia). Which assessment would best indicate to the nurse that the patient's condition is improving? a. Blood pressure (BP) is less than 140/90 mm Hg. b. Patient reports decreased exertional dyspnea. c. Heart rate is between 60 and 100 beats/minute. d. Patient's chest x-ray indicates clear lung fields.

ANS: B Because a major symptom of IPAH is exertional dyspnea, an improvement in this symptom would indicate that the medication was effective. Nifedipine will affect BP and heart rate, but these parameters would not be used to monitor the effectiveness of therapy for a patient with IPAH. The chest x-ray will show clear lung fields even if the therapy is not effective

A patient with hypertension who has just started taking atenolol (Tenormin) returns to the health clinic after 2 weeks for a follow-up visit. The blood pressure (BP) is unchanged from the previous visit. Which action should the nurse take first? a. Inform the patient about the reasons for a possible change in drug dosage. b. Question the patient about whether the medication is actually being taken. c. Inform the patient that multiple drugs are often needed to treat hypertension. d. Question the patient regarding any lifestyle changes made to help control BP.

ANS: B Because noncompliance with antihypertensive therapy is common, the nurse's initial action should be to determine whether the patient is taking the atenolol as prescribed. The other actions also may be implemented, but these would be done after assessing patient compliance with the prescribed therapy

After the nurse has received change-of-shift report, which patient should the nurse assess first? a. A patient with pneumonia who has crackles in the right lung base b. A patient with possible lung cancer who has just returned after bronchoscopy c. A patient with hemoptysis and a 16-mm induration with tuberculin skin testing d. A patient with chronic obstructive pulmonary disease (COPD) and pulmonary function testing (PFT) that indicates low forced vital capacity

ANS: B Because the cough and gag are decreased after bronchoscopy, this patient should be assessed for airway patency. The other patients do not have clinical manifestations or procedures that require immediate assessment by the nurse

The nurse identifies the nursing diagnosis of decreased cardiac output related to valvular insufficiency for the patient with infective endocarditis (IE) based on which assessment finding(s)? a. Fever, chills, and diaphoresis b. Urine output less than 30 mL/hr c. Petechiae on the inside of the mouth and conjunctiva d. Increase in heart rate of 15 beats/minute with walking

ANS: B Decreased renal perfusion caused by inadequate cardiac output will lead to decreased urine output. Petechiae, fever, chills, and diaphoresis are symptoms of IE, but are not caused by decreased cardiac output. An increase in pulse rate of 15 beats/minute is normal with exercise

An experienced nurse instructs a new nurse about how to care for a patient with dyspnea caused by a pulmonary fungal infection. Which action by the new nurse indicates a need for further teaching? a. Listening to the patient's lung sounds several times during the shift b. Placing the patient on droplet precautions and in a private hospital room c. Increasing the oxygen flow rate to keep the oxygen saturation above 90% d. Monitoring patient serology results to identify the specific infecting organism

ANS: B Fungal infections are not transmitted from person to person. Therefore no isolation procedures are necessary. The other actions by the new nurse are appropriate

Which assessment finding for a patient who is receiving IV furosemide (Lasix) to treat stage 2 hypertension is most important to report to the health care provider? a. Blood glucose level of 175 mg/dL b. Blood potassium level of 3.0 mEq/L c. Most recent blood pressure (BP) reading of 168/94 mm Hg d. Orthostatic systolic BP decrease of 12 mm Hg

ANS: B Hypokalemia is a frequent adverse effect of the loop diuretics and can cause life-threatening dysrhythmias. The health care provider should be notified of the potassium level immediately and administration of potassium supplements initiated. The elevated blood glucose and BP also indicate a need for collaborative interventions but will not require action as urgently as the hypokalemia. An orthostatic drop of 12 mm Hg is common and will require intervention only if the patient is symptomatic

After receiving report on the following patients, which patient should the nurse assess first? a. Patient with rheumatic fever who has sharp chest pain with a deep breath b. Patient with acute aortic regurgitation whose blood pressure is 86/54 mm Hg c. Patient with infective endocarditis who has a murmur and splinter hemorrhages d. Patient with dilated cardiomyopathy who has bilateral crackles at the lung bases

ANS: B Hypotension in patients with acute aortic regurgitation may indicate cardiogenic shock. The nurse should immediately assess this patient for other findings such as dyspnea or chest pain. The findings in the other patients are typical of their diagnoses and do not indicate a need for urgent assessment and intervention

A 19-year-old student comes to the student health center at the end of the semester complaining that, "My heart is skipping beats." An electrocardiogram (ECG) shows occasional premature ventricular contractions (PVCs). What action should the nurse take next? a. Start supplemental O2 at 2 to 3 L/min via nasal cannula. b. Ask the patient about current stress level and caffeine use. c. Ask the patient about any history of coronary artery disease. d. Have the patient taken to the hospital emergency department (ED).

ANS: B In a patient with a normal heart, occasional PVCs are a benign finding. The timing of the PVCs suggests stress or caffeine as possible etiologic factors. It is unlikely that the patient has coronary artery disease, and this should not be the first question the nurse asks. The patient is hemodynamically stable, so there is no indication that the patient needs to be seen in the ED or that oxygen needs to be administered

A 53-year-old patient with Stage D heart failure and type 2 diabetes asks the nurse whether heart transplant is a possible therapy. Which response by the nurse is most appropriate? a. "Because you have diabetes, you would not be a candidate for a heart transplant." b. "The choice of a patient for a heart transplant depends on many different factors." c. "Your heart failure has not reached the stage in which heart transplants are needed." d. "People who have heart transplants are at risk for multiple complications after surgery."

ANS: B Indications for a heart transplant include end-stage heart failure (Stage D), but other factors such as coping skills, family support, and patient motivation to follow the rigorous posttransplant regimen are also considered. Diabetic patients who have well-controlled blood glucose levels may be candidates for heart transplant. Although heart transplants can be associated with many complications, this response does not address the patient's question

A patient has just been admitted with probable bacterial pneumonia and sepsis. Which order should the nurse implement first? a. Chest x-ray via stretcher b. Blood cultures from two sites c. Ciprofloxacin (Cipro) 400 mg IV d. Acetaminophen (Tylenol) rectal suppository

ANS: B Initiating antibiotic therapy rapidly is essential, but it is important that the cultures be obtained before antibiotic administration. The chest x-ray and acetaminophen administration can be done last

A diabetic patient's arterial blood gas (ABG) results are pH 7.28; PaCO2 34 mm Hg; PaO2 85 mm Hg; HCO3- 18 mEq/L. The nurse would expect which finding? a. Intercostal retractions b. Kussmaul respirations c. Low oxygen saturation (SpO2) d. Decreased venous O2 pressure

ANS: B Kussmaul (deep and rapid) respirations are a compensatory mechanism for metabolic acidosis. The low pH and low bicarbonate result indicate metabolic acidosis. Intercostal retractions, a low oxygen saturation rate, and a decrease in venous O2 pressure would not be caused by acidosis

Which action is appropriate for the nurse to delegate to unlicensed assistive personnel (UAP)? a. Listen to a patient's lung sounds for wheezes or rhonchi. b. Label specimens obtained during percutaneous lung biopsy. c. Instruct a patient about how to use home spirometry testing. d. Measure induration at the site of a patient's intradermal skin test.

ANS: B Labeling of specimens is within the scope of practice of UAP. The other actions require nursing judgment and should be done by licensed nursing personnel

A patient has ST segment changes that support an acute inferior wall myocardial infarction. Which lead would be best for monitoring the patient? a. I b. II c. V2 d. V6

ANS: B Leads II, III, and AVF reflect the inferior area of the heart and the ST segment changes. Lead II will best capture any electrocardiographic (ECG) changes that indicate further damage to the myocardium. The other leads do not reflect the inferior part of the myocardial wall and will not provide data about further ischemic changes in that area

When assessing a newly admitted patient, the nurse notes a murmur along the left sternal border. To document more information about the murmur, which action will the nurse take next? a. Find the point of maximal impulse. b. Determine the timing of the murmur. c. Compare the apical and radial pulse rates. d. Palpate the quality of the peripheral pulses.

ANS: B Murmurs are caused by turbulent blood flow, such as occurs when blood flows through a damaged valve. Relevant information includes the position in which the murmur is heard best (e.g., sitting and leaning forward), the timing of the murmur in relation to the cardiac cycle (e.g., systole, diastole), and where on the thorax the murmur is heard best. The other information is also important in the cardiac assessment but will not provide information that is relevant to the murmur

During the assessment of a 25-year-old patient with infective endocarditis (IE), the nurse would expect to find a. substernal chest pressure. b. a new regurgitant murmur. c. a pruritic rash on the chest. d. involuntary muscle movement.

ANS: B New regurgitant murmurs occur in IE because vegetations on the valves prevent valve closure. Substernal chest discomfort, rashes, and involuntary muscle movement are clinical manifestations of other cardiac disorders such as angina and rheumatic fever

To assess the patient with pericarditis for evidence of a pericardial friction rub, the nurse should a. listen for a rumbling, low-pitched, systolic murmur over the left anterior chest. b. auscultate by placing the diaphragm of the stethoscope on the lower left sternal border. c. ask the patient to cough during auscultation to distinguish the sound from a pleural friction rub. d. feel the precordial area with the palm of the hand to detect vibrations with cardiac contraction.

ANS: B Pericardial friction rubs are heard best with the diaphragm at the lower left sternal border. The nurse should ask the patient to hold his or her breath during auscultation to distinguish the sounds from a pleural friction rub. Friction rubs are not typically low pitched or rumbling and are not confined to systole. Rubs are not assessed by palpation

The nurse is admitting a patient with possible rheumatic fever. Which question on the admission health history will be most pertinent to ask? a. "Do you use any illegal IV drugs?" b. "Have you had a recent sore throat?" c. "Have you injured your chest in the last few weeks?" d. "Do you have a family history of congenital heart disease?"

ANS: B Rheumatic fever occurs as a result of an abnormal immune response to a streptococcal infection. Although illicit IV drug use should be discussed with the patient before discharge, it is not a risk factor for rheumatic fever, and would not be as pertinent when admitting the patient. Family history is not a risk factor for rheumatic fever. Chest injury would cause musculoskeletal chest pain rather than rheumatic fever.

A patient with a chronic cough has a bronchoscopy. After the procedure, which intervention by the nurse is most appropriate? a. Elevate the head of the bed to 80 to 90 degrees. b. Keep the patient NPO until the gag reflex returns. c. Place on bed rest for at least 4 hours after bronchoscopy. d. Notify the health care provider about blood-tinged mucus.

ANS: B Risk for aspiration and maintaining an open airway is the priority. Because a local anesthetic is used to suppress the gag/cough reflexes during bronchoscopy, the nurse should monitor for the return of these reflexes before allowing the patient to take oral fluids or food. Blood-tinged mucus is not uncommon after bronchoscopy. The patient does not need to be on bed rest, and the head of the bed does not need to be in the high-Fowler's position

Which nursing action could the registered nurse (RN) working in a skilled care hospital unit delegate to an experienced licensed practical/vocational nurse (LPN/LVN) caring for a patient with a permanent tracheostomy? a. Assess the patient's risk for aspiration. b. Suction the tracheostomy when needed. c. Teach the patient about self-care of the tracheostomy. d. Determine the need for replacement of the tracheostomy tube.

ANS: B Suctioning of a stable patient can be delegated to LPNs/LVNs. Patient assessment and patient teaching should be done by the RN

Which assessment finding in a patient who is admitted with infective endocarditis (IE) is most important to communicate to the health care provider? a. Generalized muscle aching b. Sudden onset right flank pain c. Janeway's lesions on the palms d. Temperature 100.7° F (38.1° C)

ANS: B Sudden onset of flank pain indicates possible embolization to the kidney and may require diagnostic testing such as a renal arteriogram and interventions to improve renal perfusion. The other findings are typically found in IE, but do not require any new interventions

A patient with a history of hypertension treated with a diuretic and an angiotensin-converting enzyme (ACE) inhibitor arrives in the emergency department complaining of a severe headache and nausea and has a blood pressure (BP) of 238/118 mm Hg. Which question should the nurse ask first? a. "Did you take any acetaminophen (Tylenol) today?" b. "Have you been consistently taking your medications?" c. "Have there been any recent stressful events in your life?" d. "Have you recently taken any antihistamine medications?"

ANS: B Sudden withdrawal of antihypertensive medications can cause rebound hypertension and hypertensive crisis. Although many over-the-counter medications can cause hypertension, antihistamines and acetaminophen do not increase BP. Stressful events will increase BP but not usually to the level seen in this patient

The nurse teaches a patient about the transmission of pulmonary tuberculosis (TB). Which statement, if made by the patient, indicates that teaching was effective? a. "I will avoid being outdoors whenever possible." b. "My husband will be sleeping in the guest bedroom." c. "I will take the bus instead of driving to visit my friends." d. "I will keep the windows closed at home to contain the germs."

ANS: B Teach the patient how to minimize exposure to close contacts and household members. Homes should be well ventilated, especially the areas where the infected person spends a lot of time. While still infectious, the patient should sleep alone, spend as much time as possible outdoors, and minimize time in congregate settings or on public transportation

A registered nurse (RN) is observing a student nurse who is doing a physical assessment on a patient. The RN will need to intervene immediately if the student nurse a. presses on the skin over the tibia for 10 seconds to check for edema. b. palpates both carotid arteries simultaneously to compare pulse quality. c. documents a murmur heard along the right sternal border as a pulmonic murmur. d. places the patient in the left lateral position to check for the point of maximal impulse.

ANS: B The carotid pulses should never be palpated at the same time to avoid vagal stimulation, dysrhythmias, and decreased cerebral blood flow. The other assessment techniques also need to be corrected. However, they are not dangerous to the patient

When admitting a patient for a cardiac catheterization and coronary angiogram, which information about the patient is most important for the nurse to communicate to the health care provider? a. The patient's pedal pulses are +1. b. The patient is allergic to shellfish. c. The patient had a heart attack a year ago. d. The patient has not eaten anything today.

ANS: B The contrast dye used for the procedure is iodine based, so patients who have shellfish allergies will require treatment with medications such as corticosteroids and antihistamines before the angiogram. The other information is also communicated to the health care provider but will not require a change in the usual precardiac catheterization orders or medications

The nurse obtains the following assessment data on an older patient who has influenza. Which information will be most important for the nurse to communicate to the health care provider? a. Fever of 100.4° F (38° C) b. Diffuse crackles in the lungs c. Sore throat and frequent cough d. Myalgia and persistent headache

ANS: B The crackles indicate that the patient may be developing pneumonia, a common complication of influenza, which would require aggressive treatment. Myalgia, headache, mild temperature elevation, and sore throat with cough are typical manifestations of influenza and are treated with supportive care measures such as over-the-counter (OTC) pain relievers and increased fluid intake

Following a laryngectomy a patient coughs violently during suctioning and dislodges the tracheostomy tube. Which action should the nurse take first? a. Cover stoma with sterile gauze and ventilate through stoma. b. Attempt to reinsert the tracheostomy tube with the obturator in place. c. Assess the patient's oxygen saturation and notify the health care provider. d. Ventilate the patient with a manual bag and face mask until the health care provider arrives.

ANS: B The first action should be to attempt to reinsert the tracheostomy tube to maintain the patient's airway. Assessing the patient's oxygenation is an important action, but it is not the most appropriate first action in this situation. Covering the stoma with a dressing and manually ventilating the patient may be an appropriate action if the nurse is unable to reinsert the tracheostomy tube. Ventilating with a facemask is not appropriate for a patient with a total laryngectomy because there is a complete separation between the upper airway and the trachea.

After 2 months of tuberculosis (TB) treatment with isoniazid (INH), rifampin (Rifadin), pyrazinamide (PZA), and ethambutol, a patient continues to have positive sputum smears for acid-fast bacilli (AFB). Which action should the nurse take next? a. Teach about treatment for drug-resistant TB treatment. b. Ask the patient whether medications have been taken as directed. c. Schedule the patient for directly observed therapy three times weekly. d. Discuss with the health care provider the need for the patient to use an injectable antibiotic.

ANS: B The first action should be to determine whether the patient has been compliant with drug therapy because negative sputum smears would be expected if the TB bacillus is susceptible to the medications and if the medications have been taken correctly. Assessment is the first step in the nursing process. Depending on whether the patient has been compliant or not, different medications or directly observed therapy may be indicated. The other options are interventions based on assumptions until an assessment has been completed

Which blood pressure (BP) finding by the nurse indicates that no changes in therapy are needed for a patient with stage 1 hypertension who has a history of diabetes mellitus? a. 102/60 mm Hg b. 128/76 mm Hg c. 139/90 mm Hg d. 136/82 mm Hg

ANS: B The goal for antihypertensive therapy for a patient with hypertension and diabetes mellitus is a BP <130/80 mm Hg. The BP of 102/60 may indicate overtreatment of the hypertension and an increased risk for adverse drug effects. The other two blood pressures indicate a need for modifications in the patient's treatment

When developing a community health program to decrease the incidence of rheumatic fever, which action would be most important for the community health nurse to include? a. Vaccinate high-risk groups in the community with streptococcal vaccine. b. Teach community members to seek treatment for streptococcal pharyngitis. c. Teach about the importance of monitoring temperature when sore throats occur. d. Teach about prophylactic antibiotics to those with a family history of rheumatic fever.

ANS: B The incidence of rheumatic fever is decreased by treatment of streptococcal infections with antibiotics. Family history is not a risk factor for rheumatic fever. There is no immunization that is effective in decreasing the incidence of rheumatic fever. Teaching about monitoring temperature will not decrease the incidence of rheumatic fever.

The nurse monitors a patient after chest tube placement for a hemopneumothorax. The nurse is most concerned if which assessment finding is observed? a. A large air leak in the water-seal chamber b. 400 mL of blood in the collection chamber c. Complaint of pain with each deep inspiration d. Subcutaneous emphysema at the insertion site

ANS: B The large amount of blood may indicate that the patient is in danger of developing hypovolemic shock. An air leak would be expected immediately after chest tube placement for a pneumothorax. Initially, brisk bubbling of air occurs in this chamber when a pneumothorax is evacuated. The pain should be treated but is not as urgent a concern as the possibility of continued hemorrhage. Subcutaneous emphysema should be monitored but is not unusual in a patient with pneumothorax. A small amount of subcutaneous air is harmless and will be reabsorbed

A patient with newly diagnosed lung cancer tells the nurse, "I don't think I'm going to live to see my next birthday." Which response by the nurse is best? a. "Would you like to talk to the hospital chaplain about your feelings?" b. "Can you tell me what it is that makes you think you will die so soon?" c. "Are you afraid that the treatment for your cancer will not be effective?" d. "Do you think that taking an antidepressant medication would be helpful?"

ANS: B The nurse's initial response should be to collect more assessment data about the patient's statement. The answer beginning "Can you tell me what it is" is the most open-ended question and will offer the best opportunity for obtaining more data. The answer beginning, "Are you afraid" implies that the patient thinks that the cancer will be immediately fatal, although the patient's statement may not be related to the cancer diagnosis. The remaining two answers offer interventions that may be helpful to the patient, but more assessment is needed to determine whether these interventions are appropriate

Which action will the nurse in the hypertension clinic take in order to obtain an accurate baseline blood pressure (BP) for a new patient? a. Deflate the BP cuff at a rate of 5 to 10 mm Hg per second. b. Have the patient sit in a chair with the feet flat on the floor. c. Assist the patient to the supine position for BP measurements. d. Obtain two BP readings in the dominant arm and average the results.

ANS: B The patient should be seated with the feet flat on the floor. The BP is obtained in both arms, and the results of the two arms are not averaged. The patient does not need to be in the supine position. The cuff should be deflated at 2 to 3 mm Hg per second

After receiving the following information about four patients during change-of-shift report, which patient should the nurse assess first? a. Patient with acute pericarditis who has a pericardial friction rub b. Patient who has just returned to the unit after balloon valvuloplasty c. Patient who has hypertrophic cardiomyopathy and a heart rate of 116 d. Patient with a mitral valve replacement who has an anticoagulant scheduled

ANS: B The patient who has just arrived after balloon valvuloplasty will need assessment for complications such as bleeding and hypotension. The information about the other patients is consistent with their diagnoses and does not indicate any complications or need for urgent assessment or intervention. DIF

When assessing a patient with a sore throat, the nurse notes anterior cervical lymph node swelling, a temperature of 101.6° F (38.7° C), and yellow patches on the tonsils. Which action will the nurse anticipate taking? a. Teach the patient about the use of expectorants. b. Use a swab to obtain a sample for a rapid strep antigen test. c. Discuss the need to rinse the mouth out after using any inhalers. d. Teach the patient to avoid use of nonsteroidal antiinflammatory drugs (NSAIDs).

ANS: B The patient's clinical manifestations are consistent with streptococcal pharyngitis and the nurse will anticipate the need for a rapid strep antigen test and/or cultures. Because patients with streptococcal pharyngitis usually do not have a cough, use of expectorants will not be anticipated. Rinsing the mouth out after inhaler use may prevent fungal oral infections, but the patient's assessment data are not consistent with a fungal infection. NSAIDs are frequently prescribed for pain and fever relief with pharyngitis

A patient whose heart monitor shows sinus tachycardia, rate 132, is apneic and has no palpable pulses. What is the first action that the nurse should take? a. Perform synchronized cardioversion. b. Start cardiopulmonary resuscitation (CPR). c. Administer atropine per agency dysrhythmia protocol. d. Provide supplemental oxygen via non-rebreather mask.

ANS: B The patient's clinical manifestations indicate pulseless electrical activity and the nurse should immediately start CPR. The other actions would not be of benefit to this patient

The nurse is assessing a patient who has been admitted to the intensive care unit (ICU) with a hypertensive emergency. Which finding is most important to report to the health care provider? a. Urine output over 8 hours is 250 mL less than the fluid intake. b. The patient cannot move the left arm and leg when asked to do so. c. Tremors are noted in the fingers when the patient extends the arms. d. The patient complains of a headache with pain at level 8/10 (0 to 10 scale).

ANS: B The patient's inability to move the left arm and leg indicates that a hemorrhagic stroke may be occurring and will require immediate action to prevent further neurologic damage. The other clinical manifestations are also likely caused by the hypertension and will require rapid nursing actions, but they do not require action as urgently as the neurologic changes

A patient with rheumatic fever has subcutaneous nodules, erythema marginatum, and polyarthritis. Based on these findings, which nursing diagnosis would be most appropriate? a. Pain related to permanent joint fixation b. Activity intolerance related to arthralgia c. Risk for infection related to open skin lesions d. Risk for impaired skin integrity related to pruritus

ANS: B The patient's joint pain will lead to difficulty with activity. The skin lesions seen in rheumatic fever are not open or pruritic. Although acute joint pain will be a problem for this patient, joint inflammation is a temporary clinical manifestation of rheumatic fever and is not associated with permanent joint changes

After receiving change-of-shift report on a heart failure unit, which patient should the nurse assess first? a. Patient who is taking carvedilol (Coreg) and has a heart rate of 58 b. Patient who is taking digoxin and has a potassium level of 3.1 mEq/L c. Patient who is taking isosorbide dinitrate/hydralazine (BiDil) and has a headache d. Patient who is taking captopril (Capoten) and has a frequent nonproductive cough

ANS: B The patient's low potassium level increases the risk for digoxin toxicity and potentially fatal dysrhythmias. The nurse should assess the patient for other signs of digoxin toxicity and then notify the health care provider about the potassium level. The other patients also have side effects of their medications, but their symptoms do not indicate potentially life-threatening complications

The nurse obtains the following information from a patient newly diagnosed with prehypertension. Which finding is most important to address with the patient? a. Low dietary fiber intake b. No regular aerobic exercise c. Weight 5 pounds above ideal weight d. Drinks a beer with dinner on most nights

ANS: B The recommendations for preventing hypertension include exercising aerobically for 30 minutes most days of the week. A weight that is 5 pounds over the ideal body weight is not a risk factor for hypertension. The Dietary Approaches to Stop Hypertension (DASH) diet is high in fiber, but increasing fiber alone will not prevent hypertension from developing. The patient's alcohol intake is within guidelines and will not increase the hypertension risk

The nurse develops a plan of care to prevent aspiration in a high-risk patient. Which nursing action will be most effective? a. Turn and reposition immobile patients at least every 2 hours. b. Place patients with altered consciousness in side-lying positions. c. Monitor for respiratory symptoms in patients who are immunosuppressed. d. Insert nasogastric tube for feedings for patients with swallowing problems.

ANS: B The risk for aspiration is decreased when patients with a decreased level of consciousness are placed in a side-lying or upright position. Frequent turning prevents pooling of secretions in immobilized patients but will not decrease the risk for aspiration in patients at risk. Monitoring of parameters such as breath sounds and oxygen saturation will help detect pneumonia in immunocompromised patients, but it will not decrease the risk for aspiration. Conditions that increase the risk of aspiration include decreased level of consciousness (e.g., seizure, anesthesia, head injury, stroke, alcohol intake), difficulty swallowing, and nasogastric intubation with or without tube feeding. With loss of consciousness, the gag and cough reflexes are depressed, and aspiration is more likely to occur. Other high-risk groups are those who are seriously ill, have poor dentition, or are receiving acid-reducing medications

When planning care for a patient hospitalized with a streptococcal infective endocarditis (IE), which intervention is a priority for the nurse to include? a. Monitor labs for streptococcal antibodies. b. Arrange for placement of a long-term IV catheter. c. Teach the importance of completing all oral antibiotics. d. Encourage the patient to begin regular aerobic exercise.

ANS: B Treatment for IE involves 4 to 6 weeks of IV antibiotic therapy in order to eradicate the bacteria, which will require a long-term IV catheter such as a peripherally inserted central catheter (PICC) line. Rest periods and limiting physical activity to a moderate level are recommended during the treatment for IE. Oral antibiotics are not effective in eradicating the infective bacteria that cause IE. Blood cultures, rather than antibody levels, are used to monitor the effectiveness of antibiotic therapy

Which admission order written by the health care provider for a patient admitted with infective endocarditis (IE) and a fever would be a priority for the nurse to implement? a. Administer ceftriaxone (Rocephin) 1 g IV. b. Order blood cultures drawn from two sites. c. Give acetaminophen (Tylenol) PRN for fever. d. Arrange for a transesophageal echocardiogram.

ANS: B Treatment of the IE with antibiotics should be started as quickly as possible, but it is essential to obtain blood cultures before initiating antibiotic therapy to obtain accurate sensitivity results. The echocardiogram and acetaminophen administration also should be implemented rapidly, but the blood cultures (and then administration of the antibiotic) have the highest priority

The nurse and unlicensed assistive personnel (UAP) on the telemetry unit are caring for four patients. Which nursing action can be delegated to the UAP? a. Teaching a patient scheduled for exercise electrocardiography about the procedure b. Placing electrodes in the correct position for a patient who is to receive ECG monitoring c. Checking the catheter insertion site for a patient who is recovering from a coronary angiogram d. Monitoring a patient who has just returned to the unit after a transesophageal echocardiogram

ANS: B UAP can be educated in standardized lead placement for ECG monitoring. Assessment of patients who have had procedures where airway maintenance (transesophageal echocardiography) or bleeding (coronary angiogram) is a concern must be done by the registered nurse (RN). Patient teaching requires RN level education and scope of practice

The nurse notes that a patient's cardiac monitor shows that every other beat is earlier than expected, has no visible P wave, and has a QRS complex that is wide and bizarre in shape. How will the nurse document the rhythm? a. Ventricular couplets b. Ventricular bigeminy c. Ventricular R-on-T phenomenon d. Multifocal premature ventricular contractions

ANS: B Ventricular bigeminy describes a rhythm in which every other QRS complex is wide and bizarre looking. Pairs of wide QRS complexes are described as ventricular couplets. There is no indication that the premature ventricular contractions (PVCs) are multifocal or that the R-on-T phenomenon is occurring

A patient is scheduled for a computed tomography (CT) of the chest with contrast media. Which assessment findings should the nurse immediately report to the health care provider (select all that apply)? a. Patient is claustrophobic. b. Patient is allergic to shellfish. c. Patient recently used a bronchodilator inhaler. d. Patient is not able to remove a wedding band. e. Blood urea nitrogen (BUN) and serum creatinine levels are elevated.

ANS: B, E Because the contrast media is iodine-based and may cause dehydration and decreased renal blood flow, asking about iodine allergies (such as allergy to shellfish) and monitoring renal function before the CT scan are necessary. The other actions are not contraindications for CT of the chest, although they may be for other diagnostic tests, such as magnetic resonance imaging (MRI) or pulmonary function testing (PFT).

The nurse notes that a patient has incisional pain, a poor cough effort, and scattered rhonchi after a thoracotomy. Which action should the nurse take first? a. Assist the patient to sit upright in a chair. b. Splint the patient's chest during coughing. c. Medicate the patient with prescribed morphine. d. Observe the patient use the incentive spirometer.

ANS: C A major reason for atelectasis and poor airway clearance in patients after chest surgery is incisional pain (which increases with deep breathing and coughing). The first action by the nurse should be to medicate the patient to minimize incisional pain. The other actions are all appropriate ways to improve airway clearance but should be done after the morphine is given

A patient in metabolic alkalosis is admitted to the emergency department, and pulse oximetry (SpO2) indicates that the O2 saturation is 94%. Which action should the nurse take next? a. Administer bicarbonate. b. Complete a head-to-toe assessment. c. Place the patient on high-flow oxygen. d. Obtain repeat arterial blood gases (ABGs).

ANS: C Although the O2 saturation is adequate, the left shift in the oxyhemoglobin dissociation curve will decrease the amount of oxygen delivered to tissues, so high oxygen concentrations should be given. Bicarbonate would worsen the patient's condition. A head-to-toe assessment and repeat ABGs may be implemented. However, the priority intervention is to give high-flow oxygen

A patient who is receiving dobutamine (Dobutrex) for the treatment of acute decompensated heart failure (ADHF) has the following nursing interventions included in the plan of care. Which action will be most appropriate for the registered nurse (RN) to delegate to an experienced licensed practical/vocational nurse (LPN/LVN)? a. Assess the IV insertion site for signs of extravasation. b. Teach the patient the reasons for remaining on bed rest. c. Monitor the patient's blood pressure and heart rate every hour. d. Titrate the rate to keep the systolic blood pressure >90 mm Hg.

ANS: C An experienced LPN/LVN would be able to monitor BP and heart rate and would know to report significant changes to the RN. Teaching patients, making adjustments to the drip rate for vasoactive medications, and monitoring for serious complications such as extravasation require RN level education and scope of practice

During discharge teaching with a 68-year-old patient who had a mitral valve replacement with a mechanical valve, the nurse instructs the patient on the a. use of daily aspirin for anticoagulation. b. correct method for taking the radial pulse. c. need for frequent laboratory blood testing. d. need to avoid any physical activity for 1 month.

ANS: C Anticoagulation with warfarin (Coumadin) is needed for a patient with mechanical valves to prevent clotting on the valve. This will require frequent international normalized ratio (INR) testing. Daily aspirin use will not be effective in reducing the risk for clots on the valve. Monitoring of the radial pulse is not necessary after valve replacement. Patients should resume activities of daily living as tolerated

A patient with a tracheostomy has a new order for a fenestrated tracheostomy tube. Which action should the nurse include in the plan of care in collaboration with the speech therapist? a. Leave the tracheostomy inner cannula inserted at all times. b. Place the decannulation cap in the tube before cuff deflation. c. Assess the ability to swallow before using the fenestrated tube. d. Inflate the tracheostomy cuff during use of the fenestrated tube.

ANS: C Because the cuff is deflated when using a fenestrated tube, the patient's risk for aspiration should be assessed before changing to a fenestrated tracheostomy tube. The decannulation cap is never inserted before cuff deflation because to do so would obstruct the patient's airway. The cuff is deflated and the inner cannula removed to allow air to flow across the patient's vocal cords when using a fenestrated tube

Which action by the nurse will determine if the therapies ordered for a patient with chronic constrictive pericarditis are effective? a. Assess for the presence of a paradoxical pulse. b. Monitor for changes in the patient's sedimentation rate. c. Assess for the presence of jugular venous distention (JVD). d. Check the electrocardiogram (ECG) for ST segment changes.

ANS: C Because the most common finding on physical examination for a patient with chronic constrictive pericarditis is jugular venous distention, a decrease in JVD indicates improvement. Paradoxical pulse, ST-segment ECG changes, and changes in sedimentation rates occur with acute pericarditis but are not expected in chronic constrictive pericarditis

A patient who is on the progressive care unit develops atrial flutter, rate 150, with associated dyspnea and chest pain. Which action that is included in the hospital dysrhythmia protocol should the nurse do first? a. Obtain a 12-lead electrocardiogram (ECG). b. Notify the health care provider of the change in rhythm. c. Give supplemental O2 at 2 to 3 L/min via nasal cannula. d. Assess the patient's vital signs including oxygen saturation.

ANS: C Because this patient has dyspnea and chest pain in association with the new rhythm, the nurse's initial actions should be to address the patient's airway, breathing, and circulation (ABC) by starting with oxygen administration. The other actions also are important and should be implemented rapidly

A patient with heart failure has a new order for captopril (Capoten) 12.5 mg PO. After administering the first dose and teaching the patient about the drug, which statement by the patient indicates that teaching has been effective? a. "I will be sure to take the medication with food." b. "I will need to eat more potassium-rich foods in my diet." c. "I will call for help when I need to get up to use the bathroom." d. "I will expect to feel more short of breath for the next few days."

ANS: C Captopril can cause hypotension, especially after the initial dose, so it is important that the patient not get up out of bed without assistance until the nurse has had a chance to evaluate the effect of the first dose. The angiotensin-converting enzyme (ACE) inhibitors are potassium sparing, and the nurse should not teach the patient to purposely increase sources of dietary potassium. Increased shortness of breath is expected with the initiation of b-adrenergic blocker therapy for heart failure, not for ACE inhibitor therapy. ACE inhibitors are best absorbed when taken an hour before eating

The nurse has received the laboratory results for a patient who developed chest pain 4 hours ago and may be having a myocardial infarction. The most important laboratory result to review will be a. myoglobin. b. low-density lipoprotein (LDL) cholesterol. c. troponins T and I. d. creatine kinase-MB (CK-MB).

ANS: C Cardiac troponins start to elevate 4 to 6 hours after myocardial injury and are highly specific to myocardium. They are the preferred diagnostic marker for myocardial infarction. Myoglobin rises in response to myocardial injury within 30 to 60 minutes. It is rapidly cleared from the body, thus limiting its use in the diagnosis of myocardial infarction. LDL cholesterol is useful in assessing cardiovascular risk but is not helpful in determining whether a patient is having an acute myocardial infarction. Creatine kinase (CK-MB) is specific to myocardial injury and infarction and increases 4 to 6 hours after the infarction occurs. It is often trended with troponin levels

A patient who has a right-sided chest tube following a thoracotomy has continuous bubbling in the suction-control chamber of the collection device. Which action by the nurse is most appropriate? a. Document the presence of a large air leak. b. Notify the surgeon of a possible pneumothorax. c. Take no further action with the collection device. d. Adjust the dial on the wall regulator to decrease suction.

ANS: C Continuous bubbling is expected in the suction-control chamber and indicates that the suction-control chamber is connected to suction. An air leak would be detected in the water-seal chamber. There is no evidence of pneumothorax. Increasing or decreasing the vacuum source will not adjust the suction pressure. The amount of suction applied is regulated by the amount of water in this chamber and not by the amount of suction applied to the system

A patient has acute bronchitis with a nonproductive cough and wheezes. Which topic should the nurse plan to include in the teaching plan? a. Purpose of antibiotic therapy b. Ways to limit oral fluid intake c. Appropriate use of cough suppressants d. Safety concerns with home oxygen therapy

ANS: C Cough suppressants are frequently prescribed for acute bronchitis. Because most acute bronchitis is viral in origin, antibiotics are not prescribed unless there are systemic symptoms. Fluid intake is encouraged. Home oxygen is not prescribed for acute bronchitis, although it may be used for chronic bronchitis

The nurse will plan discharge teaching about the need for prophylactic antibiotics when having dental procedures for which patient? a. Patient admitted with a large acute myocardial infarction. b. Patient being discharged after an exacerbation of heart failure. c. Patient who had a mitral valve replacement with a mechanical valve. d. Patient being treated for rheumatic fever after a streptococcal infection.

ANS: C Current American Heart Association guidelines recommend the use of prophylactic antibiotics before dental procedures for patients with prosthetic valves to prevent infective endocarditis (IE). The other patients are not at risk for IE

A patient is diagnosed with both human immunodeficiency virus (HIV) and active tuberculosis (TB) disease. Which information obtained by the nurse is most important to communicate to the health care provider? a. The Mantoux test had an induration of 7 mm. b. The chest-x-ray showed infiltrates in the lower lobes. c. The patient is being treated with antiretrovirals for HIV infection. d. The patient has a cough that is productive of blood-tinged mucus.

ANS: C Drug interactions can occur between the antiretrovirals used to treat HIV infection and the medications used to treat TB. The other data are expected in a patient with HIV and TB.

When caring for a patient with infective endocarditis of the tricuspid valve, the nurse should monitor the patient for the development of a. flank pain. b. splenomegaly. c. shortness of breath. d. mental status changes.

ANS: C Embolization from the tricuspid valve would cause symptoms of pulmonary embolus. Flank pain, changes in mental status, and splenomegaly would be associated with embolization from the left-sided valves

The nurse completes a shift assessment on a patient admitted in the early phase of heart failure. When auscultating the patient's lungs, which finding would the nurse most likely hear? a. Continuous rumbling, snoring, or rattling sounds mainly on expiration b. Continuous high-pitched musical sounds on inspiration and expiration c. Discontinuous, high-pitched sounds of short duration heard on inspiration d. A series of long-duration, discontinuous, low-pitched sounds during inspiration

ANS: C Fine crackles are likely to be heard in the early phase of heart failure. Fine crackles are discontinuous, high-pitched sounds of short duration heard on inspiration. Rhonchi are continuous rumbling, snoring, or rattling sounds mainly on expiration. Course crackles are a series of long-duration, discontinuous, low-pitched sounds during inspiration. Wheezes are continuous high-pitched musical sounds on inspiration and expiration

The nurse teaches a patient about pulmonary function testing (PFT). Which statement, if made by the patient, indicates teaching was effective? a. "I will use my inhaler right before the test." b. "I won't eat or drink anything 8 hours before the test." c. "I should inhale deeply and blow out as hard as I can during the test." d. "My blood pressure and pulse will be checked every 15 minutes after the test."

ANS: C For PFT, the patient should inhale deeply and exhale as long, hard, and fast as possible. The other actions are not needed with PFT. The administration of inhaled bronchodilators should be avoided 6 hours before the procedure.

The nurse provides preoperative instruction for a patient scheduled for a left pneumonectomy for cancer of the lung. Which information should the nurse include about the patient's postoperative care? a. Positioning on the right side b. Bed rest for the first 24 hours c. Frequent use of an incentive spirometer d. Chest tube placement with continuous drainage

ANS: C Frequent deep breathing and coughing are needed after chest surgery to prevent atelectasis. To promote gas exchange, patients after pneumonectomy are positioned on the surgical side. Early mobilization decreases the risk for postoperative complications such as pneumonia and deep vein thrombosis. In a pneumonectomy, chest tubes may or may not be placed in the space from which the lung was removed. If a chest tube is used, it is clamped and only released by the surgeon to adjust the volume of serosanguineous fluid that will fill the space vacated by the lung. If the cavity overfills, it could compress the remaining lung and compromise the cardiovascular and pulmonary function. Daily chest x-rays can be used to assess the volume and space

An older patient has been diagnosed with possible white coat hypertension. Which action will the nurse plan to take next? a. Schedule the patient for regular blood pressure (BP) checks in the clinic. b. Instruct the patient about the need to decrease stress levels. c. Tell the patient how to self-monitor and record BPs at home. d. Inform the patient that ambulatory blood pressure monitoring will be needed.

ANS: C Having the patient self-monitor BPs at home will provide a reliable indication about whether the patient has hypertension. Regular BP checks in the clinic are likely to be high in a patient with white coat hypertension. Ambulatory blood pressure monitoring may be used if the data from self-monitoring are unclear. Although elevated stress levels may contribute to hypertension, instructing the patient about this is unlikely to reduce BP

A patient has recently started on digoxin (Lanoxin) in addition to furosemide (Lasix) and captopril (Capoten) for the management of heart failure. Which assessment finding by the home health nurse is a priority to communicate to the health care provider? a. Presence of 1 to 2+ edema in the feet and ankles b. Palpable liver edge 2 cm below the ribs on the right side c. Serum potassium level 3.0 mEq/L after 1 week of therapy d. Weight increase from 120 pounds to 122 pounds over 3 days

ANS: C Hypokalemia can predispose the patient to life-threatening dysrhythmias (e.g., premature ventricular contractions), and potentiate the actions of digoxin and increase the risk for digoxin toxicity, which can also cause life-threatening dysrhythmias. The other data indicate that the patient's heart failure requires more effective therapies, but they do not require nursing action as rapidly as the low serum potassium level

A patient has a junctional escape rhythm on the monitor. The nurse will expect the patient to have a heart rate of _____ beats/minute. a. 15 to 20 b. 20 to 40 c. 40 to 60 d. 60 to 100

ANS: C If the sinoatrial (SA) node fails to discharge, the atrioventricular (AV) node will automatically discharge at the normal rate of 40 to 60 beats/minute. The slower rates are typical of the bundle of His and the Purkinje system and may be seen with failure of both the SA and AV node to discharge. The normal SA node rate is 60 to 100 beats/minute

A patient who had a total laryngectomy has a nursing diagnosis of hopelessness related to loss of control of personal care. Which information obtained by the nurse is the best indicator that this identified problem is resolving? a. The patient lets the spouse provide tracheostomy care. b. The patient allows the nurse to suction the tracheostomy. c. The patient asks how to clean the tracheostomy stoma and tube. d. The patient uses a communication board to request "No Visitors."

ANS: C Independently caring for the laryngectomy tube indicates that the patient has regained control of personal care and hopelessness is at least partially resolved. Letting the nurse and spouse provide care and requesting no visitors may indicate that the patient is still experiencing hopelessness

The registered nurse (RN) is caring for a patient with a hypertensive crisis who is receiving sodium nitroprusside (Nipride). Which nursing action can the nurse delegate to an experienced licensed practical/vocational nurse (LPN/LVN)? a. Titrate nitroprusside to decrease mean arterial pressure (MAP) to 115 mm Hg. b. Evaluate effectiveness of nitroprusside therapy on blood pressure (BP). c. Set up the automatic blood pressure machine to take BP every 15 minutes. d. Assess the patient's environment for adverse stimuli that might increase BP.

ANS: C LPN/LVN education and scope of practice include the correct use of common equipment such as automatic blood pressure machines. The other actions require advanced nursing judgment and education, and should be done by RNs

Which action should the nurse take when administering the initial dose of oral labetalol (Normodyne) to a patient with hypertension? a. Encourage the use of hard candy to prevent dry mouth. b. Instruct the patient to ask for help if heart palpitations occur. c. Ask the patient to request assistance when getting out of bed. d. Teach the patient that headaches may occur with this medication.

ANS: C Labetalol decreases sympathetic nervous system activity by blocking both á- and b-adrenergic receptors, leading to vasodilation and a decrease in heart rate, which can cause severe orthostatic hypotension. Heart palpitations, dry mouth, dehydration, and headaches are possible side effects of other antihypertensives

The nurse observes a student who is listening to a patient's lungs who is having no problems with breathing. Which action by the student indicates a need to review respiratory assessment skills? a. The student starts at the apices of the lungs and moves to the bases. b. The student compares breath sounds from side to side avoiding bony areas. c. The student places the stethoscope over the posterior chest and listens during inspiration. d. The student instructs the patient to breathe slowly and a little more deeply than normal through the mouth.

ANS: C Listening only during inspiration indicates the student needs a review of respiratory assessment skills. At each placement of the stethoscope, listen to at least one cycle of inspiration and expiration. During chest auscultation, instruct the patient to breathe slowly and a little deeper than normal through the mouth. Auscultation should proceed from the lung apices to the bases, comparing opposite areas of the chest, unless the patient is in respiratory distress or will tire easily. If so, start at the bases (see Fig. 26-7). Place the stethoscope over lung tissue, not over bony prominences.

A 21-year-old woman is scheduled for percutaneous transluminal balloon valvuloplasty to treat mitral stenosis. Which information should the nurse include when explaining the advantages of valvuloplasty over valve replacement to the patient? a. Biologic valves will require immunosuppressive drugs after surgery. b. Mechanical mitral valves need to be replaced sooner than biologic valves. c. Lifelong anticoagulant therapy will be needed after mechanical valve replacement. d. Ongoing cardiac care by a health care provider is not necessary after valvuloplasty.

ANS: C Long-term anticoagulation therapy is needed after mechanical valve replacement, and this would restrict decisions about career and childbearing in this patient. Mechanical valves are durable and last longer than biologic valves. All valve repair procedures are palliative, not curative, and require lifelong health care. Biologic valves do not activate the immune system, and immunosuppressive therapy is not needed

Which information obtained by the nurse who is admitting the patient for magnetic resonance imaging (MRI) will be most important to report to the health care provider before the MRI? a. The patient has an allergy to shellfish. b. The patient has a history of atherosclerosis. c. The patient has a permanent ventricular pacemaker. d. The patient took all the prescribed cardiac medications today.

ANS: C MRI is contraindicated for patients with implanted metallic devices such as pacemakers. The other information also will be reported to the health care provider but does not impact on whether or not the patient can have an MRI

The nurse plans discharge teaching for a patient with chronic heart failure who has prescriptions for digoxin (Lanoxin) and hydrochlorothiazide (HydroDIURIL). Appropriate instructions for the patient include a. limit dietary sources of potassium. b. take the hydrochlorothiazide before bedtime. c. notify the health care provider if nausea develops. d. skip the digoxin if the pulse is below 60 beats/minute.

ANS: C Nausea is an indication of digoxin toxicity and should be reported so that the provider can assess the patient for toxicity and adjust the digoxin dose, if necessary. The patient will need to include potassium-containing foods in the diet to avoid hypokalemia. Patients should be taught to check their pulse daily before taking the digoxin and if the pulse is less than 60, to call their provider before taking the digoxin. Diuretics should be taken early in the day to avoid sleep disruption

A patient who is taking rifampin (Rifadin) for tuberculosis calls the clinic and reports having orange discolored urine and tears. Which is the best response by the nurse? a. Ask if the patient is experiencing shortness of breath, hives, or itching. b. Ask the patient about any visual abnormalities such as red-green color discrimination. c. Explain that orange discolored urine and tears are normal while taking this medication. d. Advise the patient to stop the drug and report the symptoms to the health care provider.

ANS: C Orange-colored body secretions are a side effect of rifampin. The patient does not have to stop taking the medication. The findings are not indicative of an allergic reaction. Alterations in red-green color discrimination commonly occurs when taking ethambutol (Myambutol), which is a different TB medication

A patient who has chronic heart failure tells the nurse, "I was fine when I went to bed, but I woke up in the middle of the night feeling like I was suffocating!" The nurse will document this assessment finding as a. orthopnea. b. pulsus alternans. c. paroxysmal nocturnal dyspnea. d. acute bilateral pleural effusion.

ANS: C Paroxysmal nocturnal dyspnea is caused by the reabsorption of fluid from dependent body areas when the patient is sleeping and is characterized by waking up suddenly with the feeling of suffocation. Pulsus alternans is the alternation of strong and weak peripheral pulses during palpation. Orthopnea indicates that the patient is unable to lie flat because of dyspnea. Pleural effusions develop over a longer time period

Which statement by a patient with restrictive cardiomyopathy indicates that the nurse's discharge teaching about self-management has been most effective? a. "I will avoid taking aspirin or other antiinflammatory drugs." b. "I will need to limit my intake of salt and fluids even in hot weather." c. "I will take antibiotics when my teeth are cleaned at the dental office." d. "I should begin an exercise program that includes things like biking or swimming."

ANS: C Patients with restrictive cardiomyopathy are at risk for infective endocarditis and should use prophylactic antibiotics for any procedure that may cause bacteremia. The other statements indicate a need for more teaching by the nurse. Dehydration and vigorous exercise impair ventricular filling in patients with restrictive cardiomyopathy. There is no need to avoid salt (unless ordered), aspirin, or NSAIDs

An occupational health nurse works at a manufacturing plant where there is potential exposure to inhaled dust. Which action, if recommended by the nurse, will be most helpful in reducing the incidence of lung disease? a. Treat workers with pulmonary fibrosis. b. Teach about symptoms of lung disease. c. Require the use of protective equipment. d. Monitor workers for coughing and wheezing.

ANS: C Prevention of lung disease requires the use of appropriate protective equipment such as masks. The other actions will help in recognition or early treatment of lung disease but will not be effective in prevention of lung damage. Repeated exposure eventually results in diffuse pulmonary fibrosis. Fibrosis is the result of tissue repair after inflammation

Which information will the nurse include when teaching a patient who is scheduled for a radiofrequency catheter ablation for treatment of atrial flutter? a. The procedure will prevent or minimize the risk for sudden cardiac death. b. The procedure will use cold therapy to stop the formation of the flutter waves. c. The procedure will use electrical energy to destroy areas of the conduction system. d. The procedure will stimulate the growth of new conduction pathways between the atria.

ANS: C Radiofrequency catheter ablation therapy uses electrical energy to "burn" or ablate areas of the conduction system as definitive treatment of atrial flutter (i.e., restore normal sinus rhythm) and tachydysrhythmias. All other statements regarding the procedure are incorrect

IV sodium nitroprusside (Nipride) is ordered for a patient with acute pulmonary edema. During the first hours of administration, the nurse will need to titrate the nitroprusside rate if the patient develops a. ventricular ectopy. b. a dry, hacking cough. c. a systolic BP <90 mm Hg. d. a heart rate <50 beats/minute.

ANS: C Sodium nitroprusside is a potent vasodilator, and the major adverse effect is severe hypotension. Coughing and bradycardia are not adverse effects of this medication. Nitroprusside does not cause increased ventricular ectopy

The health care provider writes an order for bacteriologic testing for a patient who has a positive tuberculosis skin test. Which action should the nurse take? a. Teach about the reason for the blood tests. b. Schedule an appointment for a chest x-ray. c. Teach about the need to get sputum specimens for 2 to 3 consecutive days. d. Instruct the patient to expectorate three specimens as soon as possible.

ANS: C Sputum specimens are obtained on 2 to 3 consecutive days for bacteriologic testing for M. tuberculosis. The patient should not provide all the specimens at once. Blood cultures are not used for tuberculosis testing. A chest x-ray is not bacteriologic testing. Although the findings on chest x-ray examination are important, it is not possible to make a diagnosis of TB solely based on chest x-ray findings because other diseases can mimic the appearance of TB

The nurse working on the heart failure unit knows that teaching an older female patient with newly diagnosed heart failure is effective when the patient states that a. she will take furosemide (Lasix) every day at bedtime. b. the nitroglycerin patch is applied when any chest pain develops. c. she will call the clinic if her weight goes from 124 to 128 pounds in a week. d. an additional pillow can help her sleep if she is feeling short of breath at night.

ANS: C Teaching for a patient with heart failure includes information about the need to weigh daily and notify the health care provider about an increase of 3 pounds in 2 days or 3 to 5 pounds in a week. Nitroglycerin patches are used primarily to reduce preload (not to prevent chest pain) in patients with heart failure and should be used daily, not on an "as needed" basis. Diuretics should be taken earlier in the day to avoid nocturia and sleep disturbance. The patient should call the clinic if increased orthopnea develops, rather than just compensating by further elevating the head of the bed

A patient with chronic heart failure who is taking a diuretic and an angiotensin-converting enzyme (ACE) inhibitor and who is on a low-sodium diet tells the home health nurse about a 5-pound weight gain in the last 3 days. The nurse's priority action will be to a. have the patient recall the dietary intake for the last 3 days. b. ask the patient about the use of the prescribed medications. c. assess the patient for clinical manifestations of acute heart failure. d. teach the patient about the importance of restricting dietary sodium.

ANS: C The 5-pound weight gain over 3 days indicates that the patient's chronic heart failure may be worsening. It is important that the patient be assessed immediately for other clinical manifestations of decompensation, such as lung crackles. A dietary recall to detect hidden sodium in the diet, reinforcement of sodium restrictions, and assessment of medication compliance may be appropriate interventions but are not the first nursing actions indicated.

Which assessment finding obtained by the nurse when assessing a patient with acute pericarditis should be reported immediately to the health care provider? a. Pulsus paradoxus 8 mm Hg b. Blood pressure (BP) of 168/94 c. Jugular venous distention (JVD) to jaw level d. Level 6 (0 to 10 scale) chest pain with a deep breath

ANS: C The JVD indicates that the patient may have developed cardiac tamponade and may need rapid intervention to maintain adequate cardiac output. Hypertension would not be associated with complications of pericarditis, and the BP is not high enough to indicate that there is any immediate need to call the health care provider. A pulsus paradoxus of 8 mm Hg is normal. Level 6/10 chest pain should be treated but is not unusual with pericarditis

Which topic will the nurse plan to include in discharge teaching for a patient with systolic heart failure and an ejection fraction of 33%? a. Need to begin an aerobic exercise program several times weekly b. Use of salt substitutes to replace table salt when cooking and at the table c. Benefits and side effects of angiotensin-converting enzyme (ACE) inhibitors d. Importance of making an annual appointment with the primary care provider

ANS: C The core measures for the treatment of heart failure established by The Joint Commission indicate that patients with an ejection fraction (EF) <40% receive an ACE inhibitor to decrease the progression of heart failure. Aerobic exercise may not be appropriate for a patient with this level of heart failure, salt substitutes are not usually recommended because of the risk of hyperkalemia, and the patient will need to see the primary care provider more frequently than annually

While admitting an 82-year-old with acute decompensated heart failure to the hospital, the nurse learns that the patient lives alone and sometimes confuses the "water pill" with the "heart pill." When planning for the patient's discharge the nurse will facilitate a a. consult with a psychologist. b. transfer to a long-term care facility. c. referral to a home health care agency. d. arrangements for around-the-clock care.

ANS: C The data about the patient suggest that assistance in developing a system for taking medications correctly at home is needed. A home health nurse will assess the patient's home situation and help the patient develop a method for taking the two medications as directed. There is no evidence that the patient requires services such as a psychologist consult, long-term care, or around-the-clock home care

While caring for a patient with respiratory disease, the nurse observes that the patient's SpO2 drops from 93% to 88% while the patient is ambulating in the hallway. What is the priority action of the nurse? a. Notify the health care provider. b. Document the response to exercise. c. Administer the PRN supplemental O2. d. Encourage the patient to pace activity.

ANS: C The drop in SpO2 to 85% indicates that the patient is hypoxemic and needs supplemental oxygen when exercising. The other actions are also important, but the first action should be to correct the hypoxemia

The nurse is reviewing the laboratory results for newly admitted patients on the cardiovascular unit. Which patient laboratory result is most important to communicate as soon as possible to the health care provider? a. Patient whose triglyceride level is high b. Patient who has very low homocysteine level c. Patient with increase in troponin T and troponin I level d. Patient with elevated high-sensitivity C-reactive protein level

ANS: C The elevation in troponin T and I indicates that the patient has had an acute myocardial infarction. Further assessment and interventions are indicated. The other laboratory results are indicative of increased risk for coronary artery disease but are not associated with acute cardiac problems that need immediate intervention

The nurse is caring for a patient with idiopathic pulmonary arterial hypertension (IPAH) who is receiving epoprostenol (Flolan). Which assessment information requires the most immediate action by the nurse? a. The oxygen saturation is 94%. b. The blood pressure is 98/56 mm Hg. c. The patient's central IV line is disconnected. d. The international normalized ratio (INR) is prolonged.

ANS: C The half-life of this drug is 6 minutes, so the nurse will need to restart the infusion as soon as possible to prevent rapid clinical deterioration. The other data also indicate a need for ongoing monitoring or intervention, but the priority action is to reconnect the infusion

Which action by the nurse will be most effective in decreasing the spread of pertussis in a community setting? a. Providing supportive care to patients diagnosed with pertussis b. Teaching family members about the need for careful hand washing c. Teaching patients about the need for adult pertussis immunizations d. Encouraging patients to complete the prescribed course of antibiotics

ANS: C The increased rate of pertussis in adults is thought to be due to decreasing immunity after childhood immunization. Immunization is the most effective method of protecting communities from infectious diseases. Hand washing should be taught, but pertussis is spread by droplets and contact with secretions. Supportive care does not shorten the course of the disease or the risk for transmission. Taking antibiotics as prescribed does assist with decreased transmission, but patients are likely to have already transmitted the disease by the time the diagnosis is made

While assessing a 68-year-old with ascites, the nurse also notes jugular venous distention (JVD) with the head of the patient's bed elevated 45 degrees. The nurse knows this finding indicates a. decreased fluid volume. b. jugular vein atherosclerosis. c. increased right atrial pressure. d. incompetent jugular vein valves.

ANS: C The jugular veins empty into the superior vena cava and then into the right atrium, so JVD with the patient sitting at a 45-degree angle reflects increased right atrial pressure. JVD is an indicator of excessive fluid volume (increased preload), not decreased fluid volume. JVD is not caused by incompetent jugular vein valves or atherosclerosis

During change-of-shift report, the nurse obtains the following information about a hypertensive patient who received the first dose of nadolol (Corgard) during the previous shift. Which information indicates that the patient needs immediate intervention? a. The patient's most recent blood pressure (BP) reading is 158/91 mm Hg. b. The patient's pulse has dropped from 68 to 57 beats/minute. c. The patient has developed wheezes throughout the lung fields. d. The patient complains that the fingers and toes feel quite cold.

ANS: C The most urgent concern for this patient is the wheezes, which indicate that bronchospasm (a common adverse effect of the noncardioselective b-blockers) is occurring. The nurse should immediately obtain an oxygen saturation measurement, apply supplemental oxygen, and notify the health care provider. The mild decrease in heart rate and complaint of cold fingers and toes are associated with b-receptor blockade but do not require any change in therapy. The BP reading may indicate that a change in medication type or dose may be indicated. However, this is not as urgently needed as addressing the bronchospasm

A patient with right lower-lobe pneumonia has been treated with IV antibiotics for 3 days. Which assessment data obtained by the nurse indicates that the treatment has been effective? a. Bronchial breath sounds are heard at the right base. b. The patient coughs up small amounts of green mucus. c. The patient's white blood cell (WBC) count is 9000/µL. d. Increased tactile fremitus is palpable over the right chest.

ANS: C The normal WBC count indicates that the antibiotics have been effective. All the other data suggest that a change in treatment is needed

Employee health test results reveal a tuberculosis (TB) skin test of 16-mm induration and a negative chest x-ray for a staff nurse working on the pulmonary unit. The nurse has no symptoms of TB. Which information should the occupational health nurse plan to teach the staff nurse? a. Standard four-drug therapy for TB b. Need for annual repeat TB skin testing c. Use and side effects of isoniazid (INH) d. Bacille Calmette-Guérin (BCG) vaccine

ANS: C The nurse is considered to have a latent TB infection and should be treated with INH daily for 6 to 9 months. The four-drug therapy would be appropriate if the nurse had active TB. TB skin testing is not done for individuals who have already had a positive skin test. BCG vaccine is not used in the United States for TB and would not be helpful for this individual, who already has a TB infection

The nurse discusses management of upper respiratory infections (URI) with a patient who has acute sinusitis. Which statement by the patient indicates that additional teaching is needed? a. "I can take acetaminophen (Tylenol) to treat my discomfort." b. "I will drink lots of juices and other fluids to stay well hydrated." c. "I can use my nasal decongestant spray until the congestion is all gone." d. "I will watch for changes in nasal secretions or the sputum that I cough up."

ANS: C The nurse should clarify that nasal decongestant sprays should be used for no more than 3 days to prevent rebound vasodilation and congestion. The other responses indicate that the teaching has been effective

Which action will the nurse implement for a patient who arrives for a calcium-scoring CT scan? a. Insert an IV catheter. b. Administer oral sedative medications. c. Teach the patient about the procedure. d. Confirm that the patient has been fasting.

ANS: C The nurse will need to teach the patient that the procedure is rapid and involves little risk. None of the other actions are necessary

A patient recovering from heart surgery develops pericarditis and complains of level 6 (0 to 10 scale) chest pain with deep breathing. Which ordered PRN medication will be the most appropriate for the nurse to give? a. Fentanyl 1 mg IV b. IV morphine sulfate 4 mg c. Oral ibuprofen (Motrin) 600 mg d. Oral acetaminophen (Tylenol) 650 mg

ANS: C The pain associated with pericarditis is caused by inflammation, so nonsteroidal antiinflammatory drugs (NSAIDs) (e.g., ibuprofen) are most effective. Opioid analgesics are usually not used for the pain associated with pericarditis

An outpatient who has chronic heart failure returns to the clinic after 2 weeks of therapy with metoprolol (Toprol XL). Which assessment finding is most important for the nurse to report to the health care provider? a. 2+ pedal edema b. Heart rate of 56 beats/minute c. Blood pressure (BP) of 88/42 mm Hg d. Complaints of fatigue

ANS: C The patient's BP indicates that the dose of metoprolol may need to be decreased because of hypotension. Bradycardia is a frequent adverse effect of b-adrenergic blockade, but the rate of 56 is not unusual with â-adrenergic blocker therapy. b-Adrenergic blockade initially will worsen symptoms of heart failure in many patients, and patients should be taught that some increase in symptoms, such as fatigue and edema, is expected during the initiation of therapy with this class of drugs

A 56-year-old patient who has no previous history of hypertension or other health problems suddenly develops a blood pressure (BP) of 198/110 mm Hg. After reconfirming the BP, it is appropriate for the nurse to tell the patient that a. a BP recheck should be scheduled in a few weeks. b. dietary sodium and fat content should be decreased. c. there is an immediate danger of a stroke and hospitalization will be required. d. diagnosis of a possible cause, treatment, and ongoing monitoring will be needed.

ANS: D A sudden increase in BP in a patient over age 50 with no previous hypertension history or risk factors indicates that the hypertension may be secondary to some other problem. The BP will need treatment and ongoing monitoring. If the patient has no other risk factors, a stroke in the immediate future is unlikely. There is no indication that dietary salt or fat intake have contributed to this sudden increase in BP, and reducing intake of salt and fat alone will not be adequate to reduce this BP to an acceptable level

Following an acute myocardial infarction, a previously healthy 63-year-old develops clinical manifestations of heart failure. The nurse anticipates discharge teaching will include information about a. digitalis preparations. b. b-adrenergic blockers. c. calcium channel blockers. d. angiotensin-converting enzyme (ACE) inhibitors.

ANS: D ACE inhibitor therapy is currently recommended to prevent the development of heart failure in patients who have had a myocardial infarction and as a first-line therapy for patients with chronic heart failure. Digoxin therapy for heart failure is no longer considered a first-line measure, and digoxin is added to the treatment protocol when therapy with other medications such as ACE-inhibitors, diuretics, and b-adrenergic blockers is insufficient. Calcium channel blockers are not generally used in the treatment of heart failure. The b-adrenergic blockers are not used as initial therapy for new onset heart failure

The nurse is obtaining a health history from a 24-year-old patient with hypertrophic cardiomyopathy (HC). Which information obtained by the nurse is most important? a. The patient has a history of a recent upper respiratory infection. b. The patient has a family history of coronary artery disease (CAD). c. The patient reports using cocaine a "couple of times" as a teenager. d. The patient's 29-year-old brother died from a sudden cardiac arrest.

ANS: D About half of all cases of HC have a genetic basis, and it is the most common cause of sudden cardiac death in otherwise healthy young people. The information about the patient's brother will be helpful in planning care (such as an automatic implantable cardioverter-defibrillator [AICD]) for the patient and in counseling other family members. The patient should be counseled against the use of stimulant drugs, but the limited past history indicates that the patient is not at current risk for cocaine use. Viral infections and CAD are risk factors for dilated cardiomyopathy, but not for HC

The nurse analyzes the results of a patient's arterial blood gases (ABGs). Which finding would require immediate action? a. The bicarbonate level (HCO3-) is 31 mEq/L. b. The arterial oxygen saturation (SaO2) is 92%. c. The partial pressure of CO2 in arterial blood (PaCO2) is 31 mm Hg. d. The partial pressure of oxygen in arterial blood (PaO2) is 59 mm Hg.

ANS: D All the values are abnormal, but the low PaO2 indicates that the patient is at the point on the oxyhemoglobin dissociation curve where a small change in the PaO2 will cause a large drop in the O2 saturation and a decrease in tissue oxygenation. The nurse should intervene immediately to improve the patient's oxygenation

A patient with pneumonia has a fever of 101.4° F (38.6° C), a nonproductive cough, and an oxygen saturation of 88%. The patient complains of weakness, fatigue, and needs assistance to get out of bed. Which nursing diagnosis should the nurse assign as the highest priority? a. Hyperthermia related to infectious illness b. Impaired transfer ability related to weakness c. Ineffective airway clearance related to thick secretions d. Impaired gas exchange related to respiratory congestion

ANS: D All these nursing diagnoses are appropriate for the patient, but the patient's oxygen saturation indicates that all body tissues are at risk for hypoxia unless the gas exchange is improved

A patient who has just been admitted with pulmonary edema is scheduled to receive the following medications. Which medication should the nurse question before giving? a. Furosemide (Lasix) 60 mg b. Captopril (Capoten) 25 mg c. Digoxin (Lanoxin) 0.125 mg d. Carvedilol (Coreg) 3.125 mg

ANS: D Although carvedilol is appropriate for the treatment of chronic heart failure, it is not used for patients with acute decompensated heart failure (ADHF) because of the risk of worsening the heart failure. The other medications are appropriate for the patient with ADHF

A patient arrives in the ear, nose, and throat clinic complaining of a piece of tissue being "stuck up my nose" and with foul-smelling nasal drainage from the right nare. Which action should the nurse take first? a. Notify the clinic health care provider. b. Obtain aerobic culture specimens of the drainage. c. Ask the patient about how the cotton got into the nose. d. Have the patient occlude the left nare and blow the nose.

ANS: D Because the highest priority action is to remove the foreign object from the nare, the nurse's first action should be to assist the patient to remove the object. The other actions are also appropriate but should be done after attempting to clear the nose.

When analyzing the rhythm of a patient's electrocardiogram (ECG), the nurse will need to investigate further upon finding a(n) a. isoelectric ST segment. b. P-R interval of 0.18 second. c. Q-T interval of 0.38 second. d. QRS interval of 0.14 second.

ANS: D Because the normal QRS interval is 0.04 to 0.10 seconds, the patient's QRS interval of 0.14 seconds indicates that the conduction through the ventricular conduction system is prolonged. The P-R interval and Q-T interval are within normal range, and ST segment should be isoelectric (flat).

The nurse is caring for a patient who is receiving IV furosemide (Lasix) and morphine for the treatment of acute decompensated heart failure (ADHF) with severe orthopnea. Which clinical finding is the best indicator that the treatment has been effective? a. Weight loss of 2 pounds in 24 hours b. Hourly urine output greater than 60 mL c. Reduction in patient complaints of chest pain d. Reduced dyspnea with the head of bed at 30 degrees

ANS: D Because the patient's major clinical manifestation of ADHF is orthopnea (caused by the presence of fluid in the alveoli), the best indicator that the medications are effective is a decrease in dyspnea with the head of the bed at 30 degrees. The other assessment data also may indicate that diuresis or improvement in cardiac output has occurred, but are not as specific to evaluating this patient's response

The nurse is caring for a 70-year-old who uses hydrochlorothiazide (HydroDIURIL) and enalapril (Norvasc), but whose self-monitored blood pressure (BP) continues to be elevated. Which patient information may indicate a need for a change? a. Patient takes a daily multivitamin tablet. b. Patient checks BP daily just after getting up. c. Patient drinks wine three to four times a week. d. Patient uses ibuprofen (Motrin) daily to treat osteoarthritis.

ANS: D Because use of nonsteroidal antiinflammatory drugs (NSAIDs) can prevent adequate BP control, the patient may need to avoid the use of ibuprofen. A multivitamin tablet will help supply vitamin D, which may help lower BP. BP decreases while sleeping, so self-monitoring early in the morning will result in obtaining pressures that are at their lowest. The patient's alcohol intake is not excessive

When assessing the respiratory system of an older patient, which finding indicates that the nurse should take immediate action? a. Weak cough effort b. Barrel-shaped chest c. Dry mucous membranes d. Bilateral crackles at lung bases

ANS: D Crackles in the lower half of the lungs indicate that the patient may have an acute problem such as heart failure. The nurse should immediately accomplish further assessments, such as oxygen saturation, and notify the health care provider. A barrel-shaped chest, hyperresonance to percussion, and a weak cough effort are associated with aging. Further evaluation may be needed, but immediate action is not indicated. An older patient has a less forceful cough and fewer and less functional cilia. Mucous membranes tend to be drier

The nurse obtains a health history from a 65-year-old patient with a prosthetic mitral valve who has symptoms of infective endocarditis (IE). Which question by the nurse is most appropriate? a. "Do you have a history of a heart attack?" b. "Is there a family history of endocarditis?" c. "Have you had any recent immunizations?" d. "Have you had dental work done recently?"

ANS: D Dental procedures place the patient with a prosthetic mitral valve at risk for infective endocarditis (IE). Myocardial infarction (MI), immunizations, and a family history of endocarditis are not risk factors for IE.

An alcoholic and homeless patient is diagnosed with active tuberculosis (TB). Which intervention by the nurse will be most effective in ensuring adherence with the treatment regimen? a. Arrange for a friend to administer the medication on schedule. b. Give the patient written instructions about how to take the medications. c. Teach the patient about the high risk for infecting others unless treatment is followed. d. Arrange for a daily noon meal at a community center where the drug will be administered.

ANS: D Directly observed therapy is the most effective means for ensuring compliance with the treatment regimen, and arranging a daily meal will help ensure that the patient is available to receive the medication. The other nursing interventions may be appropriate for some patients but are not likely to be as helpful for this patient

When the nurse is monitoring a patient who is undergoing exercise (stress) testing on a treadmill, which assessment finding requires the most rapid action by the nurse? a. Patient complaint of feeling tired b. Pulse change from 87 to 101 beats/minute c. Blood pressure (BP) increase from 134/68 to 150/80 mm Hg d. Newly inverted T waves on the electrocardiogram

ANS: D ECG changes associated with coronary ischemia (such as T-wave inversions and ST segment depression) indicate that the myocardium is not getting adequate oxygen delivery and that the exercise test should be terminated immediately. Increases in BP and heart rate (HR) are normal responses to aerobic exercise. Feeling tired is also normal as the intensity of exercise increases during the stress testing

A patient who has just been admitted with community-acquired pneumococcal pneumonia has a temperature of 101.6° F with a frequent cough and is complaining of severe pleuritic chest pain. Which prescribed medication should the nurse give first? a. Codeine b. Guaifenesin (Robitussin) c. Acetaminophen (Tylenol) d. Piperacillin/tazobactam (Zosyn)

ANS: D Early initiation of antibiotic therapy has been demonstrated to reduce mortality. The other medications are also appropriate and should be given as soon as possible, but the priority is to start antibiotic therapy

A patient has a normal cardiac rhythm and a heart rate of 72 beats/minute. The nurse determines that the P-R interval is 0.24 seconds. The most appropriate intervention by the nurse would be to a. notify the health care provider immediately. b. give atropine per agency dysrhythmia protocol. c. prepare the patient for temporary pacemaker insertion. d. document the finding and continue to monitor the patient.

ANS: D First-degree atrioventricular (AV) block is asymptomatic and requires ongoing monitoring because it may progress to more serious forms of heart block. The rate is normal, so there is no indication that atropine is needed. Immediate notification of the health care provider about an asymptomatic rhythm is not necessary

After the nurse teaches the patient with stage 1 hypertension about diet modifications that should be implemented, which diet choice indicates that the teaching has been effective? a. The patient avoids eating nuts or nut butters. b. The patient restricts intake of chicken and fish. c. The patient has two cups of coffee in the morning. d. The patient has a glass of low-fat milk with each meal.

ANS: D For the prevention of hypertension, the Dietary Approaches to Stop Hypertension (DASH) recommendations include increasing the intake of calcium-rich foods. Caffeine restriction and decreased protein intake are not included in the recommendations. Nuts are high in beneficial nutrients and 4 to 5 servings weekly are recommended in the DASH diet.

Which action will be included in the plan of care when the nurse is caring for a patient who is receiving nicardipine (Cardene) to treat a hypertensive emergency? a. Keep the patient NPO to prevent aspiration caused by nausea and possible vomiting. b. Organize nursing activities so that the patient has undisturbed sleep for 6 to 8 hours at night. c. Assist the patient up in the chair for meals to avoid complications associated with immobility. d. Use an automated noninvasive blood pressure machine to obtain frequent blood pressure (BP) measurements.

ANS: D Frequent monitoring of BP is needed when the patient is receiving rapid-acting IV antihypertensive medications. This can be most easily accomplished with an automated BP machine or arterial line. The patient will require frequent assessments, so allowing 6 to 8 hours of undisturbed sleep is not appropriate. When patients are receiving IV vasodilators, bed rest is maintained to prevent decreased cerebral perfusion and fainting. There is no indication that this patient is nauseated or at risk for aspiration, so an NPO status is unnecessary

Which information should the nurse include when teaching a patient with newly diagnosed hypertension? a. Increasing physical activity will control blood pressure (BP) for most patients. b. Most patients are able to control BP through dietary changes. c. Annual BP checks are needed to monitor treatment effectiveness. d. Hypertension is usually asymptomatic until target organ damage occurs.

ANS: D Hypertension is usually asymptomatic until target organ damage has occurred. Lifestyle changes (e.g., physical activity, dietary changes) are used to help manage blood pressure, but drugs are needed for most patients. Home BP monitoring should be taught to the patient and findings checked by the health care provider frequently when starting treatment for hypertension and then every 3 months once stable

A patient who has a history of chronic obstructive pulmonary disease (COPD) was hospitalized for increasing shortness of breath and chronic hypoxemia (SaO2 levels of 89% to 90%). In planning for discharge, which action by the nurse will be most effective in improving compliance with discharge teaching? a. Start giving the patient discharge teaching on the day of admission. b. Have the patient repeat the instructions immediately after teaching. c. Accomplish the patient teaching just before the scheduled discharge. d. Arrange for the patient's caregiver to be present during the teaching.

ANS: D Hypoxemia interferes with the patient's ability to learn and retain information, so having the patient's caregiver present will increase the likelihood that discharge instructions will be followed. Having the patient repeat the instructions will indicate that the information is understood at the time, but it does not guarantee retention of the information. Because the patient is likely to be distracted just before discharge, giving discharge instructions just before discharge is not ideal. The patient is likely to be anxious and even more hypoxemic than usual on the day of admission, so teaching about discharge should be postponed.

Which intervention will the nurse include in the plan of care for a patient who is diagnosed with a lung abscess? a. Teach the patient to avoid the use of over-the-counter expectorants. b. Assist the patient with chest physiotherapy and postural drainage. c. Notify the health care provider immediately about any bloody or foul-smelling sputum. d. Teach about the need for prolonged antibiotic therapy after discharge from the hospital.

ANS: D Long-term antibiotic therapy is needed for effective eradication of the infecting organisms in lung abscess. Chest physiotherapy and postural drainage are not recommended for lung abscess because they may lead to spread of the infection. Foul smelling and bloody sputum are common clinical manifestations in lung abscess. Expectorants may be used because the patient is encouraged to cough

The nurse completes discharge teaching for a patient who has had a lung transplant. The nurse evaluates that the teaching has been effective if the patient makes which statement? a. "I will make an appointment to see the doctor every year." b. "I will stop taking the prednisone if I experience a dry cough." c. "I will not worry if I feel a little short of breath with exercise." d. "I will call the health care provider right away if I develop a fever."

ANS: D Low-grade fever may indicate infection or acute rejection so the patient should notify the health care provider immediately if the temperature is elevated. Patients require frequent follow-up visits with the transplant team. Annual health care provider visits would not be sufficient. Home oxygen use is not an expectation after lung transplant. Shortness of breath should be reported. Low-grade fever, fatigue, dyspnea, dry cough, and oxygen desaturation are signs of rejection. Immunosuppressive therapy, including prednisone, needs to be continued to prevent rejection

The nurse teaches a patient about discharge instructions after a rhinoplasty. Which statement, if made by the patient, indicates that the teaching was successful? a. "I can take 800 mg ibuprofen for pain control." b. "I will safely remove and reapply nasal packing daily." c. "My nose will look normal after 24 hours when the swelling goes away." d. "I will keep my head elevated for 48 hours to minimize swelling and pain."

ANS: D Maintaining the head in an elevated position will decrease the amount of nasal swelling. NSAIDs, such as ibuprofen, increase the risk for postoperative bleeding and should not be used postoperatively. The patient would not be taught to remove or reapply nasal packing, which is usually removed by the surgeon on the day after surgery. Although return to a preinjury appearance is the goal of the surgery, it is not always possible to achieve this result, especially in the first few weeks after surgery

When teaching the patient with newly diagnosed heart failure about a 2000-mg sodium diet, the nurse explains that foods to be restricted include a. canned and frozen fruits. b. fresh or frozen vegetables. c. eggs and other high-protein foods. d. milk, yogurt, and other milk products.

ANS: D Milk and yogurt naturally contain a significant amount of sodium, and intake of these should be limited for patients on a diet that limits sodium to 2000 mg daily. Other milk products, such as processed cheeses, have very high levels of sodium and are not appropriate for a 2000-mg sodium diet. The other foods listed have minimal levels of sodium and can be eaten without restriction

A lobectomy is scheduled for a patient with stage I non-small cell lung cancer. The patient tells the nurse, "I would rather have chemotherapy than surgery." Which response by the nurse is most appropriate? a. "Are you afraid that the surgery will be very painful?" b. "Did you have bad experiences with previous surgeries?" c. "Surgery is the treatment of choice for stage I lung cancer." d. "Tell me what you know about the various treatments available."

ANS: D More assessment of the patient's concerns about surgery is indicated. An open-ended response will elicit the most information from the patient. The answer beginning, "Surgery is the treatment of choice" is accurate, but it discourages the patient from sharing concerns about surgery. The remaining two answers indicate that the nurse has jumped to conclusions about the patient's reasons for not wanting surgery. Chemotherapy is the primary treatment for small cell lung cancer. In non-small cell lung cancer, chemotherapy may be used in the treatment of nonresectable tumors or as adjuvant therapy to surgery

A patient is admitted with active tuberculosis (TB). The nurse should question a health care provider's order to discontinue airborne precautions unless which assessment finding is documented? a. Chest x-ray shows no upper lobe infiltrates. b. TB medications have been taken for 6 months. c. Mantoux testing shows an induration of 10 mm. d. Three sputum smears for acid-fast bacilli are negative.

ANS: D Negative sputum smears indicate that Mycobacterium tuberculosis is not present in the sputum, and the patient cannot transmit the bacteria by the airborne route. Chest x-rays are not used to determine whether treatment has been successful. Taking medications for 6 months is necessary, but the multidrug-resistant forms of the disease might not be eradicated after 6 months of therapy. Repeat Mantoux testing would not be done because the result will not change even with effective treatment

The nurse is caring for a patient who has acute pharyngitis caused by Candida albicans. Which action is appropriate for the nurse to include in the plan of care? a. Avoid giving patient warm liquids to drink. b. Assess patient for allergies to penicillin antibiotics. c. Teach the patient about the need to sleep in a warm, dry environment. d. Teach patient to "swish and swallow" prescribed oral nystatin (Mycostatin).

ANS: D Oral or pharyngeal fungal infections are treated with nystatin solution. The goal of the "swish and swallow" technique is to expose all of the oral mucosa to the antifungal agent. Warm liquids may be soothing to a sore throat. The patient should be taught to use a cool mist humidifier. There is no need to assess for penicillin/cephalosporin allergies because Candida albicans infection is treated with antifungals

The nurse is performing tuberculosis (TB) skin tests in a clinic that has many patients who have immigrated to the United States. Which question is most important for the nurse to ask before the skin test? a. "Is there any family history of TB?" b. "How long have you lived in the United States?" c. "Do you take any over-the-counter (OTC) medications?" d. "Have you received the bacille Calmette-Guérin (BCG) vaccine for TB?"

ANS: D Patients who have received the BCG vaccine will have a positive Mantoux test. Another method for screening (such as a chest x-ray) will need to be used in determining whether the patient has a TB infection. The other information also may be valuable but is not as pertinent to the decision about doing TB skin testing

The nurse establishes the nursing diagnosis of ineffective health maintenance related to lack of knowledge regarding long-term management of rheumatic fever when a 30-year-old recovering from rheumatic fever without carditis says which of the following? a. "I will need prophylactic antibiotic therapy for 5 years." b. "I will need to take aspirin or ibuprofen (Motrin) to relieve my joint pain." c. "I will call the doctor if I develop excessive fatigue or difficulty breathing." d. "I will be immune to further episodes of rheumatic fever after this infection."

ANS: D Patients with a history of rheumatic fever are more susceptible to a second episode. Patients with rheumatic fever without carditis require prophylaxis until age 20 and for a minimum of 5 years. The other patient statements are correct and would not support the nursing diagnosis of ineffective health maintenance

The nurse supervises unlicensed assistive personnel (UAP) who are providing care for a patient with right lower lobe pneumonia. The nurse should intervene if which action by UAP is observed? a. UAP splint the patient's chest during coughing. b. UAP assist the patient to ambulate to the bathroom. c. UAP help the patient to a bedside chair for meals. d. UAP lower the head of the patient's bed to 15 degrees.

ANS: D Positioning the patient with the head of the bed lowered will decrease ventilation. The other actions are appropriate for a patient with pneumonia

The nurse provides discharge teaching for a patient who has two fractured ribs from an automobile accident. Which statement, if made by the patient, would indicate that teaching has been effective? a. "I am going to buy a rib binder to wear during the day." b. "I can take shallow breaths to prevent my chest from hurting." c. "I should plan on taking the pain pills only at bedtime so I can sleep." d. "I will use the incentive spirometer every hour or two during the day."

ANS: D Prevention of the complications of atelectasis and pneumonia is a priority after rib fracture. This can be ensured by deep breathing and coughing. Use of a rib binder, shallow breathing, and taking pain medications only at night are likely to result in atelectasis

After noting a pulse deficit when assessing a 74-year-old patient who has just arrived in the emergency department, the nurse will anticipate that the patient may require a. emergent cardioversion. b. a cardiac catheterization. c. hourly blood pressure (BP) checks. d. electrocardiographic (ECG) monitoring.

ANS: D Pulse deficit is a difference between simultaneously obtained apical and radial pulses. It indicates that there may be a cardiac dysrhythmia that would best be detected with ECG monitoring. Frequent BP monitoring, cardiac catheterization, and emergent cardioversion are used for diagnosis and/or treatment of cardiovascular disorders but would not be as helpful in determining the immediate reason for the pulse deficit

During a physical examination of a 74-year-old patient, the nurse palpates the point of maximal impulse (PMI) in the sixth intercostal space lateral to the left midclavicular line. The most appropriate action for the nurse to take next will be to a. ask the patient about risk factors for atherosclerosis. b. document that the PMI is in the normal anatomic location. c. auscultate both the carotid arteries for the presence of a bruit. d. assess the patient for symptoms of left ventricular hypertrophy.

ANS: D The PMI should be felt at the intersection of the fifth intercostal space and the left midclavicular line. A PMI located outside these landmarks indicates possible cardiac enlargement, such as with left ventricular hypertrophy. Cardiac enlargement is not necessarily associated with atherosclerosis or carotid artery disease

The nurse obtains a rhythm strip on a patient who has had a myocardial infarction and makes the following analysis: no visible P waves, P-R interval not measurable, ventricular rate 162, R-R interval regular, and QRS complex wide and distorted, QRS duration 0.18 second. The nurse interprets the patient's cardiac rhythm as a. atrial flutter. b. sinus tachycardia. c. ventricular fibrillation. d. ventricular tachycardia.

ANS: D The absence of P waves, wide QRS, rate >150 beats/minute, and the regularity of the rhythm indicate ventricular tachycardia. Atrial flutter is usually regular, has a narrow QRS configuration, and has flutter waves present representing atrial activity. Sinus tachycardia has P waves. Ventricular fibrillation is irregular and does not have a consistent QRS duration

A patient has just been diagnosed with hypertension and has been started on captopril (Capoten). Which information is important to include when teaching the patient about this medication? a. Check blood pressure (BP) in both arms before taking the medication. b. Increase fluid intake if dryness of the mouth is a problem. c. Include high-potassium foods such as bananas in the diet. d. Change position slowly to help prevent dizziness and falls.

ANS: D The angiotensin-converting enzyme (ACE) inhibitors frequently cause orthostatic hypotension, and patients should be taught to change position slowly to allow the vascular system time to compensate for the position change. Increasing fluid intake may counteract the effect of the medication, and the patient is taught to use gum or hard candy to relieve dry mouth. The BP should be taken in the nondominant arm by newly diagnosed patients in the morning, before taking the medication, and in the evening. Because ACE inhibitors cause potassium retention, increased intake of high-potassium foods is inappropriate

A patient who was admitted with a myocardial infarction experiences a 45-second episode of ventricular tachycardia, then converts to sinus rhythm with a heart rate of 98 beats/minute. Which of the following actions should the nurse take next? a. Immediately notify the health care provider. b. Document the rhythm and continue to monitor the patient. c. Perform synchronized cardioversion per agency dysrhythmia protocol. d. Prepare to give IV amiodarone (Cordarone) per agency dysrhythmia protocol.

ANS: D The burst of sustained ventricular tachycardia indicates that the patient has significant ventricular irritability, and antidysrhythmic medication administration is needed to prevent further episodes. The nurse should notify the health care provider after the medication is started. Defibrillation is not indicated given that the patient is currently in a sinus rhythm. Documentation and continued monitoring are not adequate responses to this situation

The nurse has received change-of-shift report about the following patients on the progressive care unit. Which patient should the nurse see first? a. A patient who is in a sinus rhythm, rate 98, after having electrical cardioversion 2 hours ago b. A patient with new onset atrial fibrillation, rate 88, who has a first dose of warfarin (Coumadin) due c. A patient with second-degree atrioventricular (AV) block, type 1, rate 60, who is dizzy when ambulating d. A patient whose implantable cardioverter-defibrillator (ICD) fired two times today who has a dose of amiodarone (Cordarone) due

ANS: D The frequent firing of the ICD indicates that the patient's ventricles are very irritable, and the priority is to assess the patient and administer the amiodarone. The other patients may be seen after the amiodarone is administered

The nurse admits a patient who has a diagnosis of an acute asthma attack. Which statement indicates that the patient may need teaching regarding medication use? a. "I have not had any acute asthma attacks during the last year." b. "I became short of breath an hour before coming to the hospital." c. "I've been taking Tylenol 650 mg every 6 hours for chest-wall pain." d. "I've been using my albuterol inhaler more frequently over the last 4 days."

ANS: D The increased need for a rapid-acting bronchodilator should alert the patient that an acute attack may be imminent and that a change in therapy may be needed. The patient should be taught to contact a health care provider if this occurs. The other data do not indicate any need for additional teaching

A nurse is caring for a patient who has had a total laryngectomy and radical neck dissection. During the first 24 hours after surgery what is the priority nursing action? a. Monitor for bleeding. b. Maintain adequate IV fluid intake. c. Suction tracheostomy every eight hours. d. Keep the patient in semi-Fowler's position.

ANS: D The most important goals after a laryngectomy and radical neck dissection are to maintain the airway and ensure adequate oxygenation. Keeping the patient in a semi-Fowler's position will decrease edema and limit tension on the suture lines to help ensure an open airway. Maintenance of IV fluids and monitoring for bleeding are important, but maintaining an open airway is the priority. Tracheostomy care and suctioning should be provided as needed. During the immediate postoperative period, the patient with a laryngectomy requires frequent suctioning of the tracheostomy tube

The nurse plans to teach a patient how to manage allergic rhinitis. Which information should the nurse include in the teaching plan? a. Hand washing is the primary way to prevent spreading the condition to others. b. Use of oral antihistamines for 2 weeks before the allergy season may prevent reactions. c. Corticosteroid nasal sprays will reduce inflammation, but systemic effects limit their use. d. Identification and avoidance of environmental triggers are the best way to avoid symptoms.

ANS: D The most important intervention is to assist the patient in identifying and avoiding potential allergens. Intranasal corticosteroids (not oral antihistamines) should be started several weeks before the allergy season. Corticosteroid nasal sprays have minimal systemic absorption. Acute viral rhinitis (the common cold) can be prevented by washing hands

A patient's cardiac monitor shows sinus rhythm, rate 64. The P-R interval is 0.18 seconds at 1:00 AM, 0.22 seconds at 2:30 PM, and 0.28 seconds at 4:00 PM. Which action should the nurse take next? a. Place the transcutaneous pacemaker pads on the patient. b. Administer atropine sulfate 1 mg IV per agency dysrhythmia protocol. c. Document the patient's rhythm and assess the patient's response to the rhythm. d. Call the health care provider before giving the next dose of metoprolol (Lopressor).

ANS: D The patient has progressive first-degree atrioventricular (AV) block, and the b-blocker should be held until discussing the medication with the health care provider. Documentation and assessment are appropriate but not fully adequate responses. The patient with first-degree AV block usually is asymptomatic, and a pacemaker is not indicated. Atropine is sometimes used for symptomatic bradycardia, but there is no indication that this patient is symptomatic

A patient with a possible pulmonary embolism complains of chest pain and difficulty breathing. The nurse finds a heart rate of 142 beats/minute, blood pressure of 100/60 mmHg, and respirations of 42 breaths/minute. Which action should the nurse take first? a. Administer anticoagulant drug therapy. b. Notify the patient's health care provider. c. Prepare patient for a spiral computed tomography (CT). d. Elevate the head of the bed to a semi-Fowler's position.

ANS: D The patient has symptoms consistent with a pulmonary embolism (PE). Elevating the head of the bed will improve ventilation and gas exchange. The other actions can be accomplished after the head is elevated (and oxygen is started). A spiral CT may be ordered by the health care provider to identify PE. Anticoagulants may be ordered after confirmation of the diagnosis of PE

A patient develops sinus bradycardia at a rate of 32 beats/minute, has a blood pressure (BP) of 80/42 mm Hg, and is complaining of feeling faint. Which actions should the nurse take next? a. Recheck the heart rhythm and BP in 5 minutes. b. Have the patient perform the Valsalva maneuver. c. Give the scheduled dose of diltiazem (Cardizem). d. Apply the transcutaneous pacemaker (TCP) pads.

ANS: D The patient is experiencing symptomatic bradycardia, and treatment with TCP is appropriate. Continued monitoring of the rhythm and BP is an inadequate response. Calcium channel blockers will further decrease the heart rate, and the diltiazem should be held. The Valsalva maneuver will further decrease the rate

After providing a patient with discharge instructions on the management of a new permanent pacemaker, the nurse knows that teaching has been effective when the patient states a. "I will avoid cooking with a microwave oven or being near one in use." b. "It will be 1 month before I can take a bath or return to my usual activities." c. "I will notify the airlines when I make a reservation that I have a pacemaker." d. "I won't lift the arm on the pacemaker side up very high until I see the doctor."

ANS: D The patient is instructed to avoid lifting the arm on the pacemaker side above the shoulder to avoid displacing the pacemaker leads. The patient should notify airport security about the presence of a pacemaker before going through the metal detector, but there is no need to notify the airlines when making a reservation. Microwave oven use does not affect the pacemaker. The insertion procedure involves minor surgery that will have a short recovery period

The nurse teaches the patient being evaluated for rhythm disturbances with a Holter monitor to a. connect the recorder to a computer once daily. b. exercise more than usual while the monitor is in place. c. remove the electrodes when taking a shower or tub bath. d. keep a diary of daily activities while the monitor is worn.

ANS: D The patient is instructed to keep a diary describing daily activities while Holter monitoring is being accomplished to help correlate any rhythm disturbances with patient activities. Patients are taught that they should not take a shower or bath during Holter monitoring and that they should continue with their usual daily activities. The recorder stores the information about the patient's rhythm until the end of the testing, when it is removed and the data are analyzed

An hour after a thoracotomy, a patient complains of incisional pain at a level 7 (based on 0 to 10 scale) and has decreased left-sided breath sounds. The pleural drainage system has 100 mL of bloody drainage and a large air leak. Which action is best for the nurse to take next? a. Milk the chest tube gently to remove any clots. b. Clamp the chest tube momentarily to check for the origin of the air leak. c. Assist the patient to deep breathe, cough, and use the incentive spirometer. d. Set up the patient controlled analgesia (PCA) and administer the loading dose of morphine.

ANS: D The patient is unlikely to take deep breaths or cough until the pain level is lower. A chest tube output of 100 mL is not unusual in the first hour after thoracotomy and would not require milking of the chest tube. An air leak is expected in the initial postoperative period after thoracotomy

A patient admitted with acute dyspnea is newly diagnosed with dilated cardiomyopathy. Which information will the nurse plan to teach the patient about managing this disorder? a. A heart transplant should be scheduled as soon as possible. b. Elevating the legs above the heart will help relieve dyspnea. c. Careful compliance with diet and medications will prevent heart failure. d. Notify the doctor about any symptoms of heart failure such as shortness of breath.

ANS: D The patient should be instructed to notify the health care provider about any worsening of heart failure symptoms. Because dilated cardiomyopathy does not respond well to therapy, even patients with good compliance with therapy may have recurrent episodes of heart failure. Elevation of the legs above the heart will worsen symptoms (although this approach is appropriate for a patient with hypertrophic cardiomyopathy). The patient with terminal or end-stage cardiomyopathy may consider heart transplantation

When assessing a patient who has just arrived after an automobile accident, the emergency department nurse notes tachycardia and absent breath sounds over the right lung. For which intervention will the nurse prepare the patient? a. Emergency pericardiocentesis b. Stabilization of the chest wall with tape c. Administration of an inhaled bronchodilator d. Insertion of a chest tube with a chest drainage system

ANS: D The patient's history and absent breath sounds suggest a right-sided pneumothorax or hemothorax, which will require treatment with a chest tube and drainage. The other therapies would be appropriate for an acute asthma attack, flail chest, or cardiac tamponade, but the patient's clinical manifestations are not consistent with these problems

After change-of-shift report, which patient should the nurse assess first? a. 72-year-old with cor pulmonale who has 4+ bilateral edema in his legs and feet b. 28-year-old with a history of a lung transplant and a temperature of 101° F (38.3° C) c. 40-year-old with a pleural effusion who is complaining of severe stabbing chest pain d. 64-year-old with lung cancer and tracheal deviation after subclavian catheter insertion

ANS: D The patient's history and symptoms suggest possible tension pneumothorax, a medical emergency. The other patients also require assessment as soon as possible, but tension pneumothorax will require immediate treatment to avoid death from inadequate cardiac output or hypoxemia

When reviewing the 12-lead electrocardiograph (ECG) for a healthy 79-year-old patient who is having an annual physical examination, what will be of most concern to the nurse? a. The PR interval is 0.21 seconds. b. The QRS duration is 0.13 seconds. c. There is a right bundle-branch block. d. The heart rate (HR) is 42 beats/minute.

ANS: D The resting HR does not change with aging, so the decrease in HR requires further investigation. Bundle-branch block and slight increases in PR interval or QRS duration are common in older individuals because of increases in conduction time through the AV node, bundle of His, and bundle branches

The nurse prepares a patient with a left-sided pleural effusion for a thoracentesis. How should the nurse position the patient? a. Supine with the head of the bed elevated 30 degrees b. In a high-Fowler's position with the left arm extended c. On the right side with the left arm extended above the head d. Sitting upright with the arms supported on an over bed table

ANS: D The upright position with the arms supported increases lung expansion, allows fluid to collect at the lung bases, and expands the intercostal space so that access to the pleural space is easier. The other positions would increase the work of breathing for the patient and make it more difficult for the health care provider performing the thoracentesis

The laboratory has just called with the arterial blood gas (ABG) results on four patients. Which result is most important for the nurse to report immediately to the health care provider? a. pH 7.34, PaO2 82 mm Hg, PaCO2 40 mm Hg, and O2 sat 97% b. pH 7.35, PaO2 85 mm Hg, PaCO2 45 mm Hg, and O2 sat 95% c. pH 7.46, PaO2 90 mm Hg, PaCO2 32 mm Hg, and O2 sat 98% d. pH 7.31, PaO2 91 mm Hg, PaCO2 50 mm Hg, and O2 sat 96%

ANS: D These ABGs indicate uncompensated respiratory acidosis and should be reported to the health care provider. The other values are normal or close to normal

The nurse needs to quickly estimate the heart rate for a patient with a regular heart rhythm. Which method will be best to use? a. Count the number of large squares in the R-R interval and divide by 300. b. Print a 1-minute electrocardiogram (ECG) strip and count the number of QRS complexes. c. Calculate the number of small squares between one QRS complex and the next and divide into 1500. d. Use the 3-second markers to count the number of QRS complexes in 6 seconds and multiply by 10.

ANS: D This is the quickest way to determine the ventricular rate for a patient with a regular rhythm. All the other methods are accurate, but take longer

The nurse palpates the posterior chest while the patient says "99" and notes absent fremitus. Which action should the nurse take next? a. Palpate the anterior chest and observe for barrel chest. b. Encourage the patient to turn, cough, and deep breathe. c. Review the chest x-ray report for evidence of pneumonia. d. Auscultate anterior and posterior breath sounds bilaterally.

ANS: D To assess for tactile fremitus, the nurse should use the palms of the hands to assess for vibration when the patient repeats a word or phrase such as "99." After noting absent fremitus, the nurse should then auscultate the lungs to assess for the presence or absence of breath sounds. Absent fremitus may be noted with pneumothorax or atelectasis. The vibration is increased in conditions such as pneumonia, lung tumors, thick bronchial secretions, and pleural effusion. Turning, coughing, and deep breathing is an appropriate intervention for atelectasis, but the nurse needs to first assess breath sounds. Fremitus is decreased if the hand is farther from the lung or the lung is hyperinflated (barrel chest).The anterior of the chest is more difficult to palpate for fremitus because of the presence of large muscles and breast tissue

Which action could the nurse delegate to unlicensed assistive personnel (UAP) trained as electrocardiogram (ECG) technicians working on the cardiac unit? a. Select the best lead for monitoring a patient with an admission diagnosis of Dressler syndrome. b. Obtain a list of herbal medications used at home while admitting a new patient with pericarditis. c. Teach about the need to monitor the weight daily for a patient who has hypertrophic cardiomyopathy. d. Check the heart monitor for changes in rhythm while a patient who had a valve replacement ambulates.

ANS: D Under the supervision of registered nurses (RNs), UAP check the patient's cardiac monitor and obtain information about changes in heart rate and rhythm with exercise. Teaching and obtaining information about home medications (prescribed or complementary) and selecting the best leads for monitoring patients require more critical thinking and should be done by the RN

While caring for a 23-year-old patient with mitral valve prolapse (MVP) without valvular regurgitation, the nurse determines that discharge teaching has been effective when the patient states that it will be necessary to a. take antibiotics before any dental appointments. b. limit physical activity to avoid stressing the heart. c. take an aspirin a day to prevent clots from forming on the valve. d. avoid use of over-the-counter (OTC) medications that contain stimulant drugs.

ANS: D Use of stimulant medications should be avoided by patients with MVP because these may exacerbate symptoms. Daily aspirin and restricted physical activity are not needed by patients with mild MVP. Antibiotic prophylaxis is needed for patients with MVP with regurgitation but will not be necessary for this patient

While doing the admission assessment for a thin 76-year-old patient, the nurse observes pulsation of the abdominal aorta in the epigastric area. Which action should the nurse take? a. Teach the patient about aneurysms. b. Notify the hospital rapid response team. c. Instruct the patient to remain on bed rest. d. Document the finding in the patient chart.

ANS: D Visible pulsation of the abdominal aorta is commonly observed in the epigastric area for thin individuals. The nurse should simply document the finding in the admission assessment. Unless there are other abnormal findings (such as a bruit, pain, or hyper/hypotension) associated with the pulsation, the other actions are not necessary

A patient scheduled for a total laryngectomy and radical neck dissection for cancer of the larynx asks the nurse, "Will I be able to talk normally after surgery?" What is the best response by the nurse? a. "You will breathe through a permanent opening in your neck, but you will not be able to communicate orally." b. "You won't be able to talk right after surgery, but you will be able to speak again after the tracheostomy tube is removed." c. "You won't be able to speak as you used to, but there are artificial voice devices that will give you the ability to speak normally." d. "You will have a permanent opening into your neck, and you will need to have rehabilitation for some type of voice restoration."

ANS: D Voice rehabilitation is planned after a total laryngectomy, and a variety of assistive devices are available to restore communication. Although the ability to communicate orally is changed, it would not be appropriate to tell a patient that this ability would be lost. Artificial voice devices do not permit normal-sounding speech. In a total laryngectomy, the vocal cords are removed, so normal speech is impossible.

While assessing a patient who was admitted with heart failure, the nurse notes that the patient has jugular venous distention (JVD) when lying flat in bed. Which action should the nurse take next? a. Document this finding in the patient's record. b. Obtain vital signs, including oxygen saturation. c. Have the patient perform the Valsalva maneuver. d. Observe for JVD with the patient upright at 45 degrees.

ANS: D When the patient is lying flat, the jugular veins are at the level of the right atrium, so JVD is a common (but not a clinically significant) finding. Obtaining vital signs and oxygen saturation is not warranted at this point. JVD is an expected finding when a patient performs the Valsalva maneuver because right atrial pressure increases. JVD that persists when the patient is sitting at a 30- to 45-degree angle or greater is significant. The nurse will document the JVD in the medical record if it persists when the head is elevated

After teaching a patient with chronic stable angina about nitroglycerin, the nurse recognizes the need for further teaching when the patient makes which statement? A "I will replace my nitroglycerin supply every 6 months." B "I can take up to five tablets every 3 minutes for relief of my chest pain." C "I will take acetaminophen (Tylenol) to treat the headache caused by nitroglycerin." D "I will take the nitroglycerin 10 minutes before planned activity that usually causes chest pain."

B "I can take up to five tablets every 3 minutes for relief of my chest pain." The recommended dose of nitroglycerin is one tablet taken sublingually (SL) or one metered spray for symptoms of angina. If symptoms are unchanged or worse after 5 minutes, the patient should be instructed to activate the emergency medical services (EMS) system. If symptoms are improved, repeat the nitroglycerin every 5 minutes for a maximum of three doses and contact EMS if symptoms have not resolved completely.

Which patient is at greatest risk for sudden cardiac death? A A 42-year-old white woman with hypertension and dyslipidemia B A 52-year-old African American man with left ventricular failure C A 62-year-old obese man with diabetes mellitus and high cholesterol D A 72-year-old Native American woman with a family history of heart disease

B A 52-year-old African American man with left ventricular failure Patients with left ventricular dysfunction (ejection fraction < 30%) and ventricular dysrhythmias after MI are at greatest risk for sudden cardiac death (SCD). Other risk factors for SCD include (1) male gender (especially African American men), (2) family history of premature atherosclerosis, (3) tobacco use, (4) diabetes mellitus, (5) hypercholesterolemia, (6) hypertension, and (7) cardiomyopathy.

Which individuals would the nurse identify as having the highest risk for CAD? A A 45-year-old depressed male with a high-stress job B A 60-year-old male with below normal homocysteine levels C A 54-year-old female vegetarian with elevated high-density lipoprotein (HDL) levels D A 62-year-old female who has a sedentary lifestyle and body mass index (BMI) of 23 kg/m2

B A 60-year-old male with below normal homocysteine levels The 45-year-old depressed male with a high-stress job is at the highest risk for CAD. Studies demonstrate that depression and stressful states can contribute to the development of CAD. Elevated HDL levels and low homocysteine levels actually help to prevent CAD. Although a sedentary lifestyle is a risk factor, a BMI of 23 kg/m2 depicts normal weight, and thus the patient with two risk factors is at greatest risk for developing CAD.

The patient comes to the ED with severe, prolonged angina that is not immediately reversible. The nurse knows that if the patient once had angina related to a stable atherosclerotic plaque and the plaque ruptures, there may be occlusion of a coronary vessel and this type of pain. How will the nurse document this situation related to pathophysiology, presentation, diagnosis, prognosis, and interventions for this disorder? A Unstable angina B Acute coronary syndrome (ACS) C ST-segment-elevation myocardial infarction (STEMI) D Non-ST-segment-elevation myocardial infarction (NSTEMI)

B Acute coronary syndrome (ACS) The pain with ACS is severe, prolonged, and not easy to relieve. ACS is associated with deterioration of a once-stable atherosclerotic plaque that ruptures, exposes the intima to blood, and stimulates platelet aggregation and local vasoconstriction with thrombus formation. The unstable lesion, if partially occlusive, will be manifest as unstable angina or NSTEMI. If there is total occlusion, it is manifest as a STEMI.

The nurse would assess a patient with complaints of chest pain for which clinical manifestations associated with a myocardial infarction (MI) (select all that apply)? A Flushing B Ashen skin C Diaphoresis D Nausea and vomiting E S3 or S4 heart sounds

B Ashen skin C Diaphoresis D Nausea and vomiting E S3 or S4 heart sounds During the initial phase of an MI, catecholamines are released from the ischemic myocardial cells, causing increased sympathetic nervous system (SNS) stimulation. This results in the release of glycogen, diaphoresis, and vasoconstriction of peripheral blood vessels. The patient's skin may be ashen, cool, and clammy (not flushed) as a result of this response. Nausea and vomiting may result from reflex stimulation of the vomiting center by severe pain. Ventricular dysfunction resulting from the MI may lead to the presence of the abnormal S3 and S4 heart sounds.

The nurse is providing teaching to a patient recovering from an MI. How should resumption of sexual activity be discussed? A Delegated to the primary care provider B Discussed along with other physical activities C Avoided because it is embarrassing to the patient D Accomplished by providing the patient with written material

B Discussed along with other physical activities Although some nurses may not feel comfortable discussing sexual activity with patients, it is a necessary component of patient teaching. It is helpful to consider sex as a physical activity and to discuss or explore feelings in this area when other physical activities are discussed. Although providing the patient with written material is appropriate, it should not replace a verbal dialogue that can address the individual patient's questions and concerns.

The nurse prepares a discharge teaching plan for a 44-year-old male patient who has recently been diagnosed with coronary artery disease. Which risk factor should the nurse plan to focus on during the teaching session? A Type A personality B Elevated serum lipids C Family cardiac history D Hyperhomocysteinemia

B Elevated serum lipids Dyslipidemia is one of the four major modifiable risk factors for coronary artery disease (CAD). The other major modifiable risk factors are hypertension, tobacco use, and physical inactivity. Research findings related to psychologic states (i.e., type A personality) as a risk factor for coronary artery disease have been inconsistent. Family history is a nonmodifiable risk factor. High homocysteine levels have been linked to an increased risk for CAD.

Which antilipemic medications should the nurse question for a patient with cirrhosis of the liver (select all that apply)? A Niacin (Nicobid) B Ezetimibe (Zetia) C Gemfibrozil (Lopid) D Atorvastatin (Lipitor) E Cholestyramine (Questran)

B Ezetimibe (Zetia) D Atorvastatin (Lipitor) Ezetimibe (Zetia) should not be used by patients with liver impairment. Adverse effects of atorvastatin (Lipitor), a statin drug, include liver damage and myopathy. Liver enzymes must be monitored frequently and the medication stopped if these enzymes increase. Niacin's side effects subside with time, although decreased liver function may occur with high doses. Cholestyramine is safe for long-term use.

A patient is admitted to the hospital with possible acute pericarditis. The nurse should plan to teach the patient about the purpose of a. echocardiography. b. daily blood cultures. c. cardiac catheterization. d. 24-hour Holter monitor.

ANS: A Echocardiograms are useful in detecting the presence of the pericardial effusions associated with pericarditis. Blood cultures are not indicated unless the patient has evidence of sepsis. Cardiac catheterization and 24-hour Holter monitor is not a diagnostic procedure for pericarditis

To determine whether there is a delay in impulse conduction through the atria, the nurse will measure the duration of the patient's a. P wave. b. Q wave. c. P-R interval. d. QRS complex.

ANS: A The P wave represents the depolarization of the atria. The P-R interval represents depolarization of the atria, atrioventricular (AV) node, bundle of His, bundle branches, and the Purkinje fibers. The QRS represents ventricular depolarization. The Q wave is the first negative deflection following the P wave and should be narrow and short

The nurse is reviewing the laboratory test results for a patient who has recently been diagnosed with hypertension. Which result is most important to communicate to the health care provider? a. Serum creatinine of 2.8 mg/dL b. Serum potassium of 4.5 mEq/L c. Serum hemoglobin of 14.7 g/dL d. Blood glucose level of 96 mg/dL

ANS: A The elevated creatinine indicates renal damage caused by the hypertension. The other laboratory results are normal

After receiving change-of-shift report on a heart failure unit, which patient should the nurse assess first? a. A patient who is cool and clammy, with new-onset confusion and restlessness b. A patient who has crackles bilaterally in the lung bases and is receiving oxygen. c. A patient who had dizziness after receiving the first dose of captopril (Capoten) d. A patient who is receiving IV nesiritide (Natrecor) and has a blood pressure of 100/62

ANS: A The patient who has "wet-cold" clinical manifestations of heart failure is perfusing inadequately and needs rapid assessment and changes in management. The other patients also should be assessed as quickly as possible but do not have indications of severe decreases in tissue perfusion.

The nurse on the intermediate care unit received change-of-shift report on four patients with hypertension. Which patient should the nurse assess first? a. 43-year-old with a (blood pressure (BP) of 160/92 who is complaining of chest pain b. 52-year-old with a BP of 212/90 who has intermittent claudication c. 50-year-old with a BP of 190/104 who has a creatinine of 1.7 mg/dL d. 48-year-old with a BP of 172/98 whose urine shows microalbuminuria

ANS: A The patient with chest pain may be experiencing acute myocardial infarction, and rapid assessment and intervention are needed. The symptoms of the other patients also show target organ damage but are not indicative of acute processes

Which factors will the nurse consider when calculating the CURB-65 score for a patient with pneumonia (select all that apply)? a. Age b. Blood pressure c. Respiratory rate d. Oxygen saturation e. Presence of confusion f. Blood urea nitrogen (BUN) level

ANS: A, B, C, E, F Data collected for the CURB-65 are mental status (confusion), BUN (elevated), blood pressure (decreased), respiratory rate (increased), and age (65 and older). The other information is also essential to assess, but are not used for CURB-65 scoring

The nurse is assessing a patient with myocarditis before administering the scheduled dose of digoxin (Lanoxin). Which finding is most important for the nurse to communicate to the health care provider? a. Leukocytosis b. Irregular pulse c. Generalized myalgia d. Complaint of fatigue

ANS: B Myocarditis predisposes the heart to digoxin-associated dysrhythmias and toxicity. The other findings are common symptoms of myocarditis and there is no urgent need to report these.

A patient is admitted to the emergency department with an open stab wound to the left chest. What is the first action that the nurse should take? a. Position the patient so that the left chest is dependent. b. Tape a nonporous dressing on three sides over the chest wound. c. Cover the sucking chest wound firmly with an occlusive dressing. d. Keep the head of the patient's bed at no more than 30 degrees elevation.

ANS: B The dressing taped on three sides will allow air to escape when intrapleural pressure increases during expiration, but it will prevent air from moving into the pleural space during inspiration. Placing the patient on the left side or covering the chest wound with an occlusive dressing will allow trapped air in the pleural space and cause tension pneumothorax. The head of the bed should be elevated to 30 to 45 degrees to facilitate breathing

A transesophageal echocardiogram (TEE) is ordered for a patient with possible endocarditis. Which action included in the standard TEE orders will the nurse need to accomplish first? a. Start an IV line. b. Place the patient on NPO status. c. Administer O2 per nasal cannula. d. Give lorazepam (Ativan) 1 mg IV.

ANS: B The patient will need to be NPO for 6 hours preceding the TEE, so the nurse should place the patient on NPO status as soon as the order is received. The other actions also will need to be accomplished but not until just before or during the procedure

Which diagnostic test will be most useful to the nurse in determining whether a patient admitted with acute shortness of breath has heart failure? a. Serum troponin b. Arterial blood gases c. B-type natriuretic peptide d. 12-lead electrocardiogram

ANS: C B-type natriuretic peptide (BNP) is secreted when ventricular pressures increase, as they do with heart failure. Elevated BNP indicates a probable or very probable diagnosis of heart failure. A twelve-lead electrocardiogram, arterial blood gases, and troponin may also be used in determining the causes or effects of heart failure but are not as clearly diagnostic of heart failure as BNP

When caring for a patient with mitral valve stenosis, it is most important that the nurse assess for a. diastolic murmur. b. peripheral edema. c. shortness of breath on exertion. d. right upper quadrant tenderness.

ANS: C The pressure gradient changes in mitral stenosis lead to fluid backup into the lungs, resulting in hypoxemia and dyspnea. The other findings also may be associated with mitral valve disease but are not indicators of possible hypoxemia

To determine the effects of therapy for a patient who is being treated for heart failure, which laboratory result will the nurse plan to review? a. Troponin b. Homocysteine (Hcy) c. Low-density lipoprotein (LDL) d. B-type natriuretic peptide (BNP)

ANS: D Increased levels of BNP are a marker for heart failure. The other laboratory results would be used to assess for myocardial infarction (troponin) or risk for coronary artery disease (Hcy and LDL).

The nurse has identified a nursing diagnosis of acute pain related to inflammatory process for a patient with acute pericarditis. The priority intervention by the nurse for this problem is to a. teach the patient to take deep, slow breaths to control the pain. b. force fluids to 3000 mL/day to decrease fever and inflammation. c. remind the patient to request opioid pain medication every 4 hours. d. place the patient in Fowler's position, leaning forward on the overbed table.

ANS: D Sitting upright and leaning forward frequently will decrease the pain associated with pericarditis. Forcing fluids will not decrease the inflammation or pain. Taking deep breaths will tend to increase pericardial pain. Opioids are not very effective at controlling pain caused by acute inflammatory conditions and are usually ordered PRN. The patient would receive scheduled doses of a nonsteroidal antiinflammatory drug (NSAID).

The nurse is examining the ECG of a patient who has just been admitted with a suspected MI. Which ECG change is most indicative of prolonged or complete coronary occlusion? A Sinus tachycardia B Pathologic Q wave C Fibrillatory P waves D Prolonged PR interval

B Pathologic Q wave The presence of a pathologic Q wave, as often accompanies STEMI, is indicative of complete coronary occlusion. Sinus tachycardia, fibrillatory P waves (e.g., atrial fibrillation), or a prolonged PR interval (first-degree heart block) are not direct indicators of extensive occlusion.

The nurse recognizes that additional teaching is needed when the patient with asthma says? A. I should exercise every day if my symptoms are controlled B. I may use OTC bronchodilator drugs occasionally if I develop chest tightness C. I should inform my spouse about my medications and how to get help if I have a sever asthma attack D. A diary to record my medication use, symptoms, peak expiratory flow rates, and activity levels will help in adjusting my therapy

B. I may use OTC bronchodilator drugs occasionally if I develop chest tightness

The nurse evaluates that nursing interventions to promote airway clearance in a patient admitted with COPD are successful based on which finding? A. Absence of dyspnea B. Improved mental status C. Effective and productive coughing D. PaO2 within normal range for the patient

C. Effective and productive coughing Airway clearance is most directly evaluated as successful if the patient can engage in effective and productive coughing. Absence of dyspnea, improved mental status, and PaO2 within normal range for the patient show improved respiratory status but do not evaluate airway clearance.

Which medication is dry powder inhaler that is used only for COPD? A. Roflumilast (Daliresp) B. Salmeterol (Serevent) C. Ipratropium (Atrovent HFA) D. Indacterol (Arcapta Neohaler)

D. Indacterol (Arcapta Neohaler)

The nurse is teaching a patient how to self-administer ipratropium (Atrovent) via a metered dose inhaler (MDI). Which instruction given by the nurse is most appropriate to help the patient learn the proper inhalation technique? A. "Avoid shaking the inhaler before use." B. "Breathe out slowly before positioning the inhaler." C. "Using a spacer should be avoided for this type of medication." D. "After taking a puff, hold the breath for 30 seconds before exhaling."

B. "Breathe out slowly before positioning the inhaler." It is important to breathe out slowly before positioning the inhaler. This allows the patient to take a deeper breath while inhaling the medication, thus enhancing the effectiveness of the dose. The inhaler should be shaken well. A spacer may be used. Holding the breath after the inhalation of medication helps keep the medication in the lungs, but 30 seconds will not be possible for a patient with COPD.

A patient is admitted to the emergency department with an acute asthma attack. Which assessment is of greatest concern to the nurse? A. The presence of pulus paradoxus B. Markedly diminished breath sounds with no wheezing C. A RR of 34 and increased pulse and BP D. Use of accessory muscles of respiration and a feeling of suffocation

B. Markedly diminished breath sounds with no wheezing

When caring for a patient with chronic obstructive pulmonary disease (COPD), the nurse identifies a nursing diagnosis of imbalanced nutrition: less than body requirements after noting a weight loss of 30 lb. Which intervention should the nurse add to the plan of care for this patient? A. Order fruits and fruit juices to be offered between meals B. Order a high-calorie, high-protein diet with six small meals a day C. Teach the patient to use frozen meals at home that can be microwaved D. Provide a high-calorie, high-carbohydrate, nonirritating, frequent feeding diet

B. Order a high-calorie, high-protein diet with six small meals a day Because the patient with COPD needs to use greater energy to breathe, there is often decreased oral intake because of dyspnea. A full stomach also impairs the ability of the diaphragm to descend during inspiration, thus interfering with the work of breathing. For these reasons, the patient with COPD should eat six small meals per day taking in a high-calorie, high-protein diet, with non-protein calories divided evenly between fat and carbohydrate. The other interventions will not increase the patient's caloric intake.

The nurse determines that a patient is experiencing common adverse effects from the inhaled corticosteroid beclomethasone (Beclovent) after what occurs? A. Hypertension and pulmonary edema B. Oropharyngeal candidiasis and hoarseness C. Elevation of blood glucose and calcium levels D. Adrenocortical dysfunction and hyperglycemia

B. Oropharyngeal candidiasis and hoarseness

What is an indication of marked bronchoconstriciton with air trapping and hyperinflation of the lungs in a patient with asthma? A. SaO2 of 88% B. PEF rate of < 150 L/min C. FEV1 of 85% of predicted D. Chest x-ray showing a flattened diaphragm

B. PEF rate of < 150 L/min

Postoperative care of a patient undergoing coronary artery bypass graft (CABG) surgery includes monitoring for what common complication? A Dehydration B Paralytic ileus C Atrial dysrhythmias D Acute respiratory distress syndrome

C Atrial dysrhythmias Postoperative dysrhythmias, specifically atrial dysrhythmias, are common in the first 3 days following CABG surgery. Although the other complications could occur, they are not common complications.

The nurse assesses the right femoral artery puncture site as soon as the patient arrives after having a stent inserted into a coronary artery. The insertion site is not bleeding or discolored. What should the nurse do next to ensure the femoral artery is intact? A Palpate the insertion site for induration. B Assess peripheral pulses in the right leg. C Inspect the patient's right side and back. D Compare the color of the left and right legs.

C Inspect the patient's right side and back. The best method to determine that the right femoral artery is intact after inspection of the insertion site is to logroll the patient to inspect the right side and back for retroperitoneal bleeding. The artery can be leaking and blood is drawn into the tissues by gravity. The peripheral pulses, color, and sensation of the right leg will be assessed per agency protocol.

A 74-year-old man with a history of prostate cancer and hypertension is admitted to the emergency department with substernal chest pain. Which action will the nurse complete before administering sublingual nitroglycerin? A Administer morphine sulfate IV. B Auscultate heart and lung sounds. C Obtain a 12-lead electrocardiogram (ECG). D Assess for coronary artery disease risk factors.

C Obtain a 12-lead electrocardiogram (ECG). If a patient has chest pain, the nurse should institute the following measures: (1) administer supplemental oxygen and position the patient in upright position unless contraindicated, (2) assess vital signs, (3) obtain a 12-lead ECG, (4) provide prompt pain relief first with a nitrate followed by an opioid analgesic if needed, and (5) auscultate heart sounds. Obtaining a 12-lead ECG during chest pain aids in the diagnosis.

When teaching the patient with chronic obstructive pulmonary disease (COPD) about smoking cessation, what information should be included related to the effects of smoking on the lungs and the increased incidence of pulmonary infections? A. Smoking causes a hoarse voice. B. Cough will become nonproductive. C. Decreased alveolar macrophage function D. Sense of smell is decreased with smoking

C. Decreased alveolar macrophage function The damage to the lungs includes alveolar macrophage dysfunction that increases the incidence of infections and thus increases patient discomfort and cost to treat the infections. Other lung damage that contributes to infections includes cilia paralysis or destruction, increased mucus secretion, and bronchospasms that lead to sputum accumulation and increased cough. The patient may already be aware of respiratory mucosa damage with hoarseness and decreased sense of smell and taste, but these do not increase the incidence of pulmonary infection.

The major advantage of a Venturi mask is that it can? A. Deliver up to 80% O2 B. Provide continuous O2 humidity C. Deliver a precise concentration of O2 D. Be used while a patient eats and sleeps

C. Deliver a precise concentration of O2

A patient with asthma has the following arterial blood gas results early in an acute asthma attack pH 7.48 PaCo2- 30 PaO2-78 What is the most appropriate action by the nurse? A. Prepare the patient for mechanical ventilation B. Have the patient breathe into a paper bag to raise the PaCO2 C. Document the findings and monitor the ABG for a trend toward alkalosis D. Reduce the patient's oxygen flow rate to keep the PaO2 at the current level

C. Document the findings and monitor the ABG for a trend toward alkalosis

The nurse is assigned to care for a patient who has anxiety and an exacerbation of asthma. What is the primary reason for the nurse to carefully inspect the chest wall of this patient? A. Allow time to calm the patient B. Observe for signs of diaphoresis C. Evaluate the use of intercostal muscles D. Monitor the patient for bilateral chest expansion

C. Evaluate the use of intercostal muscles

Nursing assessment findings of jugular venous distention and pedal edema would be indicative of what complication of chronic obstructive pulmonary disease (COPD)? A. Acute respiratory failure B. Secondary respiratory infection C. Fluid volume excess resulting from cor pulmonale D. Pulmonary edema caused by left-sided heart failure

C. Fluid volume excess resulting from cor pulmonale Cor pulmonale is a right-sided heart failure caused by resistance to right ventricular outflow resulting from lung disease. With failure of the right ventricle, the blood emptying into the right atrium and ventricle would be slowed, leading to jugular venous distention and pedal edema.

Which medications are the most effective in improving asthma control by reducing bronchial hyperresponsiveness blocking the late phase reaction, and inhibiting migration of inflammatory cells? Select all that apply. A. Zileuton (Zyflo CR) B. Omalizumaub (Xolar) C. Fluticasone (Flovent HFA) D. Salmeterol (Serevent) E. Montelukast (Singular) F. Budesonide G. Beclomethasone (Qvar) H. Theophylline (Elixophyllin) I. Mometasone (Asmanex Twisthaler)

C. Fluticasone (Flovent HFA) F. Budesonide G. Beclomethasone (Qvar) I. Mometasone (Asmanex Twisthaler)

The nurse teaches pursed lip breathing to a patient who is newly diagnosed with chronic obstructive pulmonary disease (COPD). The nurse reinforces that this technique will assist respiration by which mechanism? A. Loosening secretions so that they may be coughed up more easily B. Promoting maximal inhalation for better oxygenation of the lungs C. Preventing bronchial collapse and air trapping in the lungs during exhalation D. Increasing the respiratory rate and giving the patient control of respiratory patterns

C. Preventing bronchial collapse and air trapping in the lungs during exhalation The purpose of pursed lip breathing is to slow down the exhalation phase of respiration, which decreases bronchial collapse and subsequent air trapping in the lungs during exhalation. It does not affect secretions, inhalation, or increase the rate of breathing.

What should the nurse include when teaching a patient with COPD about the need for physical exercise? A. All patients with COPD should be able to increase walking gradually to 20 minutes/day B. A bronchodilator inhaler should be used to relieve exercise induced dyspnea immediately after exercise C. Shortness of breath is expected during exercise but should return to baseline within 5 minutes D. Monitoring the HR before and after exercise is the best way to determine how much exercise can be tolerated

C. Shortness of breath is expected during exercise but should return to baseline within 5 minutes

The nurse is caring for a 48-year-old male patient admitted for exacerbation of chronic obstructive pulmonary disease (COPD). The patient develops severe dyspnea at rest, with a change in respiratory rate from 26 breaths/minute to 44 breaths/minute. Which action by the nurse would be the most appropriate? A. Have the patient perform huff coughing B. Perform chest physiotherapy for 5 minutes C. Teach the patient to use pursed-lip breathing D. Instruct the patient in diaphragmatic breathing

C. Teach the patient to use pursed-lip breathing

The nurse instructs a 68-year-old woman with hypercholesterolemia about natural lipid-lowering therapies. The nurse determines further teaching is necessary if the patient makes which statement? A "Omega-3 fatty acids are helpful in reducing triglyceride levels." B "I should check with my physician before I start taking any herbal products." C "Herbal products do not go through as extensive testing as prescription drugs do." D "I will take garlic instead of my prescription medication to reduce my cholesterol."

D "I will take garlic instead of my prescription medication to reduce my cholesterol." Current evidence does not support using garlic in the treatment of elevated cholesterol. Strong evidence supports the use of omega-3 fatty acids for reduction of triglyceride levels. Many herbal products are not standardized and effects are not predictable. Patients should consult with their health care provider before starting herbal or natural therapies.

The patient is being dismissed from the hospital after ACS and will be attending rehabilitation. What information does the patient need to be taught about the early recovery phase of rehabilitation? A Therapeutic lifestyle changes should become lifelong habits. B Physical activity is always started in the hospital and continued at home. C Attention will focus on management of chest pain, anxiety, dysrhythmias, and other complications. D Activity level is gradually increased under cardiac rehabilitation team supervision and with ECG monitoring.

D Activity level is gradually increased under cardiac rehabilitation team supervision and with ECG monitoring. In the early recovery phase after the patient is dismissed from the hospital, the activity level is gradually increased under supervision and with ECG monitoring. The late recovery phase includes therapeutic lifestyle changes that become lifelong habits. In the first phase of recovery, activity is dependent on the severity of the angina or MI, and attention is focused on the management of chest pain, anxiety, dysrhythmias, and other complications. With early recovery phase, the cardiac rehabilitation team may suggest that physical activity be initiated at home, but this is not always done.

For which problem is percutaneous coronary intervention (PCI) most clearly indicated? A Chronic stable angina B Left-sided heart failure C Coronary artery disease D Acute myocardial infarction

D Acute myocardial infarction PCI is indicated to restore coronary perfusion in cases of myocardial infarction. Chronic stable angina and CAD are normally treated with more conservative measures initially. PCI is not relevant to the pathophysiology of heart failure, such as left-sided heart failure.

When evaluating a patient's knowledge regarding a low-sodium, low-fat cardiac diet, the nurse recognizes additional teaching is needed when the patient selects which food choice? A Baked flounder B Angel food cake C Baked potato with margarine D Canned chicken noodle soup

D Canned chicken noodle soup Canned soups are very high in sodium content. Patients need to be taught to read food labels for sodium and fat content.

A patient was admitted to the emergency department (ED) 24 hours earlier with complaints of chest pain that were subsequently attributed to ST-segment-elevation myocardial infarction (STEMI). What complication of MI should the nurse anticipate? A Unstable angina B Cardiac tamponade C Sudden cardiac death D Cardiac dysrhythmias

D Cardiac dysrhythmias The most common complication after MI is dysrhythmias, which are present in 80% of patients. Unstable angina is considered a precursor to MI rather than a complication. Cardiac tamponade is a rare event, and sudden cardiac death is defined as an unexpected death from cardiac causes. Cardiac dysfunction in the period following an MI would not be characterized as sudden cardiac death.

A patient experienced sudden cardiac death (SCD) and survived. What should the nurse expect to be used as preventive treatment for the patient? A External pacemaker B An electrophysiologic study (EPS) C Medications to prevent dysrhythmias D Implantable cardioverter-defibrillator (ICD)

D Implantable cardioverter-defibrillator (ICD) An ICD is the most common approach to preventing recurrence of SCD. An external pacemaker may be used in the hospital but will not be used for the patient living daily life at home. An EPS may be done to determine if a recurrence is likely and determine the most effective medication treatment. Medications to prevent dysrhythmias are used but are not the best prevention of SCD.

Which statement made by the patient with chronic obstructive pulmonary disease (COPD) indicates a need for further teaching regarding the use of an ipratropium inhaler? A. "I can rinse my mouth following the two puffs to get rid of the bad taste." B. "I should wait at least 1 to 2 minutes between each puff of the inhaler." C. "Because this medication is not fast-acting, I cannot use it in an emergency if my breathing gets worse." D. "If my breathing gets worse, I should keep taking extra puffs of the inhaler until I can breathe more easily."

D. "If my breathing gets worse, I should keep taking extra puffs of the inhaler until I can breathe more easily." The patient should not just keep taking extra puffs of the inhaler to make breathing easier. Excessive treatment could trigger paradoxical bronchospasm, which would worsen the patient's respiratory status. Rinsing the mouth after the puffs will eliminate a bad taste. Waiting 1 to 2 minutes between each puff will facilitate the effectiveness of the administration. Ipratropium is not used in an emergency for COPD.

When teaching the patient about going from a metered dose inhaler to a dry powder inhaler,which statement by the patient shows the nurse that the patient needs more teaching? A. I do not need to use the spacer like I used to B. I will hold my breath for 10 seconds or longer if I can C. I will not shake this inhaler like I did my older inhaler D. I will store it in the bathroom so I will be able to clean it when I need to

D. I will store it in the bathroom so I will be able to clean it when I need to

The nurse supervises a team including another registered nurse (RN), a licensed practical/vocational nurse (LPN/LVN), and unlicensed assistive personnel (UAP) on a medical unit. The team is caring for many patients with respiratory problems. In what situation should the nurse intervene with teaching for a team member? A. LPN/LVN obtained a pulse oximetry reading of 94% but did not report it B. RN taught the patient about home oxygen safety in preparation for discharge C. UAP report to the nurse that the patient is complaining of difficulty breathing D. LPN/LVN changed the type of oxygen device based on arterial blood gas results

D. LPN/LVN changed the type of oxygen device based on arterial blood gas results

In addition to smoking cessation, what treatment is included for COPD to slow the progression of the disease? A. Use of bronchodilator drugs B. Use of inhaled corticosteroids C. Lung volume reduction surgery D. Prevention of respiratory tract infections

D. Prevention of respiratory tract infections

Which of the following guidelines would be a part of teaching patients how to use a metered-dose inhaler (MDI)? A. After activating the MDI, breathe in as quickly as you can B. Estimate the remaining amount of medicine in the MDI by floating the canister in water C.Disassemble the plastic canister from the inhaler and rinse both pieces under running water every week D. To determine how long the canister will last, divide the total number of puffs in the canister by the puffs needed per day

D. To determine how long the canister will last, divide the total number of puffs in the canister by the puffs needed per day

While assisting a patient with intermittent asthma to identify specific triggers of asthma, what should the nurse explain? A. Food and drug allergies do not manifest in respiratory symptoms B. Exercise induced asthma is seen only in individuals with sensitivity to cold air C. Asthma attacks are psychogenic in origin and can be controlled with relaxation techniques D. Viral upper respiratory infections are a common precipitating factor in acute asthma attacks

D. Viral upper respiratory infections are a common precipitating factor in acute asthma attacks

Before discharge, the nurse discusses activity levels with a 61-year-old patient with chronic obstructive pulmonary disease (COPD) and pneumonia. Which exercise goal is most appropriate once the patient is fully recovered from this episode of illness? A. Slightly increase activity over the current level B. Swim for 10 min/day, gradually increasing to 30 min/day C. Limit exercise to activities of daily living to conserve energy D. Walk for 20 min/day, keeping the pulse rate less than 130 beats/min

D. Walk for 20 min/day, keeping the pulse rate less than 130 beats/min The patient will benefit from mild aerobic exercise that does not stress the cardiorespiratory system. The patient should be encouraged to walk for 20 min/day, keeping the pulse rate less than 75% to 80% of maximum heart rate (220 - patient's age).

The physician has prescribed salmeterol (Serevent) for a patient with asthma. In reviewing the use of dry powder inhalers (DPIs) with the patient, what instructions should the nurse provide? A. "Close lips tightly around the mouthpiece and breathe in deeply and quickly." B. "To administer a DPI, you must use a spacer that holds the medicine so that you can inhale it." C. "You will know you have correctly used the DPI when you taste or sense the medicine going into your lungs." D. "Hold the inhaler several inches in front of your mouth and breathe in slowly, holding the medicine as long as possible."

A. "Close lips tightly around the mouthpiece and breathe in deeply and quickly."

The patient has an order for each of the following inhalers. Which one should the nurse offer to the patient at the onset of an asthma attack? A. Albuterol (Proventil) B. Salmeterol (Serevent) C. Beclomethasone (Qvar) D. Ipratropium bromide (Atrovent)

A. Albuterol (Proventil) Albuterol is a short-acting bronchodilator that should be given initially when the patient experiences an asthma attack. Salmeterol (Serevent) is a long-acting β2-adrenergic agonist, which is not used for acute asthma attacks. Beclomethasone (Qvar) is a corticosteroid inhaler and not recommended for an acute asthma attack. Ipratropium bromide (Atrovent) is an anticholinergic agent that is less effective than β2-adrenergic agonists. It may be used in an emergency with a patient unable to tolerate short-acting β2-adrenergic agonists (SABAs).

A patient is concerned that he may have asthma. Of the symptoms that he relates to the nurse, which ones suggest asthma or risk factors for asthma? Select all that apply A. Allergic rhinitis B. Prolonged inhalation C. History of skin allergies D. Cough, especially at night E. Gastric reflux or heartburn

A. Allergic rhinitis C. History of skin allergies D. Cough, especially at night E. Gastric reflux or heartburn

A 45-year-old man with asthma is brought to the emergency department by automobile. He is short of breath and appears frightened. During the initial nursing assessment, which clinical manifestation might be present as an early manifestation during an exacerbation of asthma? A. Anxiety B. Cyanosis C. Bradycardia D. Hypercapnia

A. Anxiety

A male patient with COPD becomes dyspneic at rest. His baseline blood gas results are PaO2 70 mm Hg, PaCO2 52 mm Hg, and pH 7.34. What updated patient assessment requires the nurse's priority intervention? A. Arterial pH 7.26 B. PaCO2 50 mm Hg C. Patient in tripod position D. Increased sputum expectoration

A. Arterial pH 7.26 The patient's pH shows acidosis that supports an exacerbation of COPD along with the worsening dyspnea. The PaCO2 has improved from baseline, the tripod position helps the patient's breathing, and the increase in sputum expectoration will improve the patient's ventilation.

The nurse is caring for a patient with COPD. Which intervention could be delegated to a CNA? A. Assisting the patient to get out of bed B. Auscultate breath sounds every 4 hours C. Plan patient activities to minimize exertion D. Teach the patient pursed lip breathing

A. Assisting the patient to get out of bed

A barrel chest can be seen in? Select all that apply. A. Chronic obstructive lung disease B. Normal aging C. Tension pneumothorax D. Short-term oxygen therapy

A. Chronic obstructive lung disease B. Normal aging

The nurse determines that the patient is NOT experiencing adverse effects of albuterol (Proventil) after noting which patient vital sign? A. Pulse rate of 72/minute B. Temperature of 98.4° F C. Oxygen saturation 96% D. Respiratory rate of 18/minute

A. Pulse rate of 72/minute Albuterol is a β2-agonist that can sometimes cause adverse cardiovascular effects. These would include tachycardia and angina. A pulse rate of 72 indicates that the patient did not experience tachycardia as an adverse effect.

Which method of oxygen administration is the safest system to use or a patient with COPD? A. Venturi mask B. Nasal cannula C. Simple face mask D. Non-rebreather mask

A. Venturi mask

The nurse is caring for a patient with an acute exacerbation of asthma. Following initial treatment, what finding indicates to the nurse that the patient's respiratory status is improving? A. Wheezing becomes louder B. Cough remains nonproductive C. Vesicular breath sounds decrease D. Aerosol bronchodilators stimulate coughing

A. Wheezing becomes louder The primary problem during an exacerbation of asthma is narrowing of the airway and subsequent diminished air exchange. As the airways begin to dilate, wheezing gets louder because of better air exchange. Vesicular breath sounds will increase with improved respiratory status. After a severe asthma exacerbation, the cough may be productive and stringy. Coughing after aerosol bronchodilators may indicate a problem with the inhaler or its use.

The nurse identifies the nursing diagnosis of activity intolerance for a patient with asthma. In patients with asthma, the nurse assesses for which etiologic factor for this nursing diagnosis? A. Work of breathing B. Fear of suffocation C. Effects of medications D. Anxiety and restlessness

A. Work of breathing

Clinical manifestations of COPD include? Select all that apply. A. weight loss B. barrel chest C. polycythemia D. cor pulmonale E. persistent cough F. flattened diaphragm G. decreased breath sounds H. increased total lung capacity I. frequent sputum production

A. weight loss B. barrel chest C. polycythemia D. cor pulmonale E. persistent cough F. flattened diaphragm G. decreased breath sounds H. increased total lung capacity I. frequent sputum production

Clinical manifestations of Asthma include? Select all that apply. A. wheezing B. weight loss C. barrel chest D. polycythemia E. cor pulmonale F. flattened diaphragm G. frequent sputum production H. increased fractional exhaled nitric oxide

A. wheezing B. weight loss C. barrel chest D. polycythemia E. cor pulmonale F. flattened diaphragm G. frequent sputum production H. increased fractional exhaled nitric oxide

The nurse obtains a blood pressure of 176/83 mm Hg for a patient. What is the patient's mean arterial pressure (MAP)?

ANS: 114 mm Hg MAP = (SBP + 2 DBP)/3

When analyzing an electrocardiographic (ECG) rhythm strip of a patient with a regular heart rhythm, the nurse counts 30 small blocks from one R wave to the next. The nurse calculates the patient's heart rate as ____.

ANS: 50 There are 1500 small blocks in a minute, and the nurse will divide 1500 by 30.

Which assessment finding in a patient admitted with acute decompensated heart failure (ADHF) requires the most immediate action by the nurse? a. Oxygen saturation of 88% b. Weight gain of 1 kg (2.2 lb) c. Heart rate of 106 beats/minute d. Urine output of 50 mL over 2 hours

ANS: A A decrease in oxygen saturation to less than 92% indicates hypoxemia. The nurse should administer supplemental oxygen immediately to the patient. An increase in apical pulse rate, 1-kg weight gain, and decreases in urine output also indicate worsening heart failure and require nursing actions, but the low oxygen saturation rate requires the most immediate nursing action

Following assessment of a patient with pneumonia, the nurse identifies a nursing diagnosis of ineffective airway clearance. Which assessment data best supports this diagnosis? a. Weak, nonproductive cough effort b. Large amounts of greenish sputum c. Respiratory rate of 28 breaths/minute d. Resting pulse oximetry (SpO2) of 85%

ANS: A The weak, nonproductive cough indicates that the patient is unable to clear the airway effectively. The other data would be used to support diagnoses such as impaired gas exchange and ineffective breathing pattern

The nurse receives change-of-shift report on the following four patients. Which patient should the nurse assess first? a. A 23-year-old patient with cystic fibrosis who has pulmonary function testing scheduled b. A 46-year-old patient on bed rest who is complaining of sudden onset of shortness of breath c. A 77-year-old patient with tuberculosis (TB) who has four antitubercular medications due in 15 minutes d. A 35-year-old patient who was admitted the previous day with pneumonia and has a temperature of 100.2° F (37.8° C)

ANS: B Patients on bed rest who are immobile are at high risk for deep vein thrombosis (DVT). Sudden onset of shortness of breath in a patient with a DVT suggests a pulmonary embolism and requires immediate assessment and action such as oxygen administration. The other patients should also be assessed as soon as possible, but there is no indication that they may need immediate action to prevent clinical deterioration

The nurse provides discharge instructions to a patient who was hospitalized for pneumonia. Which statement, if made by the patient, indicates a good understanding of the instructions? a. "I will call the doctor if I still feel tired after a week." b. "I will continue to do the deep breathing and coughing exercises at home." c. "I will schedule two appointments for the pneumonia and influenza vaccines." d. "I'll cancel my chest x-ray appointment if I'm feeling better in a couple weeks."

ANS: B Patients should continue to cough and deep breathe after discharge. Fatigue is expected for several weeks. The Pneumovax and influenza vaccines can be given at the same time in different arms. Explain that a follow-up chest x-ray needs to be done in 6 to 8 weeks to evaluate resolution of pneumonia

Which action should the nurse perform when preparing a patient with supraventricular tachycardia for cardioversion who is alert and has a blood pressure of 110/66 mm Hg? a. Turn the synchronizer switch to the "off" position. b. Give a sedative before cardioversion is implemented. c. Set the defibrillator/cardioverter energy to 360 joules. d. Provide assisted ventilations with a bag-valve-mask device.

ANS: B When a patient has a nonemergency cardioversion, sedation is used just before the procedure. The synchronizer switch is turned "on" for cardioversion. The initial level of joules for cardioversion is low (e.g., 50). Assisted ventilations are not indicated for this patient

A patient with acute shortness of breath is admitted to the hospital. Which action should the nurse take during the initial assessment of the patient? a. Ask the patient to lie down to complete a full physical assessment. b. Briefly ask specific questions about this episode of respiratory distress. c. Complete the admission database to check for allergies before treatment. d. Delay the physical assessment to first complete pulmonary function tests.

ANS: B When a patient has severe respiratory distress, only information pertinent to the current episode is obtained, and a more thorough assessment is deferred until later. Obtaining a comprehensive health history or full physical examination is unnecessary until the acute distress has resolved. Brief questioning and a focused physical assessment should be done rapidly to help determine the cause of the distress and suggest treatment. Checking for allergies is important, but it is not appropriate to complete the entire admission database at this time. The initial respiratory assessment must be completed before any diagnostic tests or interventions can be ordered

A patient with a pleural effusion is scheduled for a thoracentesis. Which action should the nurse take to prepare the patient for the procedure? a. Start a peripheral IV line to administer the necessary sedative drugs. b. Position the patient sitting upright on the edge of the bed and leaning forward. c. Obtain a large collection device to hold 2 to 3 liters of pleural fluid at one time. d. Remove the water pitcher and remind the patient not to eat or drink anything for 6 hours.

ANS: B When the patient is sitting up, fluid accumulates in the pleural space at the lung bases and can more easily be located and removed. The patient does not usually require sedation for the procedure, and there are no restrictions on oral intake because the patient is not sedated or unconscious. Usually only 1000 to 1200 mL of pleural fluid is removed at one time. Rapid removal of a large volume can result in hypotension, hypoxemia, or pulmonary edema

A nurse who is caring for patient with a tracheostomy tube in place has just auscultated rhonchi bilaterally. If the patient is unsuccessful in coughing up secretions, what action should the nurse take? a. Encourage increased incentive spirometer use. b. Encourage the patient to increase oral fluid intake. c. Put on sterile gloves and use a sterile catheter to suction. d. Preoxygenate the patient for 3 minutes before suctioning.

ANS: C This patient needs suctioning now to secure a patent airway. Sterile gloves and a sterile catheter are used when suctioning a tracheostomy. Preoxygenation for 3 minutes is not necessary. Incentive spirometer (IS) use opens alveoli and can induce coughing, which can mobilize secretions. However, the patient with a tracheostomy may not be able to use an incentive spirometer. Increasing oral fluid intake would not moisten and help mobilize secretions in a timely manner

A patient with a history of chronic heart failure is admitted to the emergency department (ED) with severe dyspnea and a dry, hacking cough. Which action should the nurse do first? a. Auscultate the abdomen. b. Check the capillary refill. c. Auscultate the breath sounds. d. Assess the level of orientation.

ANS: C This patient's severe dyspnea and cough indicate that acute decompensated heart failure (ADHF) is occurring. ADHF usually manifests as pulmonary edema, which should be detected and treated immediately to prevent ongoing hypoxemia and cardiac/respiratory arrest. The other assessments will provide useful data about the patient's volume status and also should be accomplished rapidly, but detection (and treatment) of pulmonary complications is the priority

Which nursing action can the registered nurse (RN) delegate to experienced unlicensed assistive personnel (UAP) working as a telemetry technician on the cardiac care unit? a. Decide whether a patient's heart rate of 116 requires urgent treatment. b. Monitor a patient's level of consciousness during synchronized cardioversion. c. Observe cardiac rhythms for multiple patients who have telemetry monitoring. d. Select the best lead for monitoring a patient admitted with acute coronary syndrome.

ANS: C UAP serving as telemetry technicians can monitor cardiac rhythms for individuals or groups of patients. Nursing actions such as assessment and choice of the most appropriate lead based on ST segment elevation location require RN-level education and scope of practice

Which laboratory result for a patient with multifocal premature ventricular contractions (PVCs) is most important for the nurse to communicate to the health care provider? a. Blood glucose 243 mg/dL b. Serum chloride 92 mEq/L c. Serum sodium 134 mEq/L d. Serum potassium 2.9 mEq/L

ANS: D Hypokalemia increases the risk for ventricular dysrhythmias such as PVCs, ventricular tachycardia, and ventricular fibrillation. The health care provider will need to prescribe a potassium infusion to correct this abnormality. Although the other laboratory values also are abnormal, they are not likely to be the etiology of the patient's PVCs and do not require immediate correction

The nurse teaches a 33-year-old male patient with asthma how to administer fluticasone (Flovent HFA) by metered-dose inhaler (MDI). Which statement by the patient to the nurse indicates correct understanding of the instructions? A. "I should not use a spacer device with this inhaler." B. "I will rinse my mouth each time after I use this inhaler." C. "I will feel my breathing improve over the next 2 to 3 hours." D. "I should use this inhaler immediately if I have trouble breathing."

B. "I will rinse my mouth each time after I use this inhaler."

The nurse determines that the patient understood medication instructions about the use of a spacer device when taking inhaled medications after hearing the patient state what as the primary benefit? A. "I will pay less for medication because it will last longer." B. "More of the medication will get down into my lungs to help my breathing." C. "Now I will not need to breathe in as deeply when taking the inhaler medications." D. "This device will make it so much easier and faster to take my inhaled medications."

B. "More of the medication will get down into my lungs to help my breathing."

Which test result identifies that a patient with asthma is responding to treatment? A. An increase in CO2 levels B. A decreased exhaled nitric oxide C. A decrease in white blood cell count D. An increase in serum bicarbonate levels

B. A decreased exhaled nitric oxide Nitric oxide levels are increased in the breath of people with asthma. A decrease in the exhaled nitric oxide concentration suggests that the treatment may be decreasing the lung inflammation associated with asthma and adherence to treatment. An increase in CO2 levels, decreased white blood cell count, and increased serum bicarbonate levels do not indicate a positive response to treatment in the asthma patient.

Which dietary modification helps to meet the nutritional needs of patients with COPD? A. Eating a high carb, low fat diet B. Avoiding foods that require a lot of chewing C. Preparing most foods of the diet to be eaten hot D. Drinking fluids with meals to promote digestion

B. Avoiding foods that require a lot of chewing

Adventitious breath sounds that do not clear with coughing and reflect fluid deep within the alveoli are? A. Pleural friction rubs B. Crackles C. Rhonchi D. Wheezes

B. Crackles

The nurse, who has administered a first dose of oral prednisone to a patient with asthma, writes on the care plan to begin monitoring for which patient parameters? A. Apical pulse B. Daily weight C. Bowel sounds D. Deep tendon reflexes

B. Daily weight Corticosteroids such as prednisone can lead to weight gain. For this reason, it is important to monitor the patient's daily weight. The drug should not affect the apical pulse, bowel sounds, or deep tendon reflexes.

A client with asthma is treated for acute exacerbation in the emergency room. A nurse reports which of the following knowing that it is NOT an indication that the condition is improving? A. Increased wheezing B. Decreased wheezing C. Warm, dry skin D. A pulse rate of 80 beats per minute

B. Decreased wheezing May indicate increase in bronchoconstriction or severe, widespread airflow obstruction

When planning teaching for the patient with chronic obstructive pulmonary disease (COPD), the nurse understands that what causes the manifestations of the disease? A. An overproduction of the antiprotease α1-antitrypsin B. Hyperinflation of alveoli and destruction of alveolar walls C. Hypertrophy and hyperplasia of goblet cells in the bronchi D. Collapse and hypoventilation of the terminal respiratory unit

B. Hyperinflation of alveoli and destruction of alveolar walls In COPD there are structural changes that include hyperinflation of alveoli, destruction of alveolar walls, destruction of alveolar capillary walls, narrowing of small airways, and loss of lung elasticity. An autosomal recessive deficiency of antitrypsin may cause COPD. Not all patients with COPD have excess mucus production by the increased number of goblet cells.

The nurse is evaluating if a patient understands how to safely determine whether a metered dose inhaler (MDI) is empty. The nurse interprets that the patient understands this important information to prevent medication underdosing when the patient describes which method to check the inhaler? A. Place it in water to see if it floats B. Keep track of the number of inhalations used C. Shake the canister while holding it next to the ear D. Check the indicator line on the side of the canister

B. Keep track of the number of inhalations used It is no longer appropriate to see if a canister floats in water or not since this is not an accurate way to determine the remaining inhaler doses. The best method to determine when to replace an inhaler is by knowing the maximum puffs available per MDI and then replacing it after the number of days when those inhalations have been used. (100 puffs/2 puffs each day = 50 days)

To decrease the patients sense of panic during an acute asthma attack, what is the best action for the nurse to do? A. Leave the patient alone to rest in a quiet, calm environment B. Stay with the patient and encourage slow, pursed lip breathing C. Reassure the patient that the attack can be controlled with treatment D. Let the patient know that frequent monitoring is being done using measurement of vital signs and SpO2

B. Stay with the patient and encourage slow, pursed lip breathing

The patient has had COPD for years and his ABGs usually show hypoxia (PaO2 <60 or SaO2 <88%) and hypercapnia (PaCO2>45). Which ABG results show movement toward respiratory acidosis and further hypoxia indicating respiratory failure? A. pH 7.35, PaO2 62, PaCO2 45 B. pH 7.34, PaO2 45, PaCO2 65 C. pH 7.42 PaO2 90, PaCO2 43 D. pH 7.46, PaO2 92, PaCO2 32

B. pH 7.34, PaO2 45, PaCO2 65

In caring for the patient with angina, the patient said, "I walked to the bathroom. While I was having a bowel movement, I started having the worst chest pain ever, like before I was admitted. I called for a nurse, but the pain is gone now." What further assessment data should the nurse obtain from the patient? A "What precipitated the pain?" B "Has the pain changed this time?" C "In what areas did you feel this pain?" D "Rate the pain on a scale from 0 to 10, with 0 being no pain and 10 being the worst pain you can imagine."

C "In what areas did you feel this pain?" Using PQRST, the assessment data not volunteered by the patient is the radiation of pain, the area the patient felt the pain, and if it radiated. The precipitating event was going to the bathroom and having a bowel movement. The quality of the pain was "like before I was admitted," although a more specific description may be helpful. Severity of the pain was the "worst chest pain ever," although an actual number may be needed. Timing is supplied by the patient describing when the pain occurred and that he had previously had this pain.

What does the nurse include when planning for postural drainage for the patient with COPD? A. Schedule the procedure 1 hour before and after meals B. Have the patient cough before positioning to clear the lungs C. Assesses the patients tolerance for dependent (head-down) positions D. Ensures that percussion and vibration are performed before positioning the patient

C. Assesses the patients tolerance for dependent (head-down) positions

When teaching the patient with mild asthma about the use of the peak flowmeter, what should the nurse instruct the patient to do? A. Carry the flowmeter with the patient at all times in case an asthma attack occurs B. Use the flowmeter to check the status of the patient's asthma every time the patient takes quick relief medication C. Follow the written asthma action plan if the expiratory flow rate is in the yellow zone D. Use the flowmeter by emptying the lungs, closing the mouth around the mouthpiece, and inhaling through the meter as quickly as possible

C. Follow the written asthma action plan if the expiratory flow rate is in the yellow zone

Which position is most appropriate for the nurse to place a patient experiencing an asthma exacerbation? A. Supine B. Lithotomy C. High Fowler's D. Reverse Trendelenburg

C. High Fowler's

The nurse evaluates that a patient is experiencing the expected beneficial effects of ipratropium (Atrovent) after noting which assessment finding? A. Decreased respiratory rate B. Increased respiratory rate C. Increased peak flow readings D. Decreased sputum production

C. Increased peak flow readings Ipratropium is a bronchodilator that should result in increased peak expiratory flow rates (PEFRs)

During an assessment of a 45-year-old patient with asthma, the nurse notes wheezing and dyspnea. The nurse interprets that these symptoms are related to what pathophysiologic change? A. Laryngospasm B. Pulmonary edema C. Narrowing of the airway D. Overdistention of the alveoli

C. Narrowing of the airway

Which breathing technique should the nurse teach the patient with moderate COPD to promote exhalation? A. Huff coughing B. Thoracic breathing C. Pursed lip breathing D. Diaphragmatic breathing

C. Pursed lip breathing

The nurse is assigned to care for a patient in the emergency department admitted with an exacerbation of asthma. The patient has received a β-adrenergic bronchodilator and supplemental oxygen. If the patient's condition does not improve, the nurse should anticipate what as the most likely next step in treatment? A. IV fluids B. Biofeedback therapy C. Systemic corticosteroids D. Pulmonary function testing

C. Systemic corticosteroids

A female patient who has type 1 diabetes mellitus has chronic stable angina that is controlled with rest. She states that over the past few months she has required increasing amounts of insulin. What goal should the nurse use to plan care that should help prevent cardiovascular disease progression? A Exercise almost every day. B Avoid saturated fat intake. C Limit calories to daily limit. D Keep Hgb A1C less than 7%.

D Keep Hgb A1C less than 7%. If the Hgb A1C is kept below 7%, this means that the patient has had good control of her blood glucose over the past 3 months. The patient indicates that increasing amounts of insulin are being required to control her blood glucose. This patient may not be adhering to the dietary guidelines or therapeutic regimen, so teaching about how to maintain diet, exercise, and medications to maintain stable blood glucose levels will be needed to achieve this goal.

A male patient who has coronary artery disease (CAD) has serum lipid values of LDL cholesterol 98 mg/dL and HDL cholesterol 47 mg/dL. What should the nurse include in the patient teaching? A Consume a diet low in fats. B Reduce total caloric intake. C Increase intake of olive oil. D The lipid levels are normal.

D The lipid levels are normal. For men, the recommended LDL is less than 100 mg/dL, and the recommended level for HDL is greater than 40mg/dL. His normal lipid levels should be included in the patient teaching and encourage him to continue taking care of himself. Assessing his need for teaching related to diet should also be done.

A patient has been receiving oxygen per nasal cannula while hospitalized for COPD. The patient asks the nurse whether oxygen use will be needed at home. What is the most appropriate response by the nurse? A. "Long-term home oxygen therapy should be used to prevent respiratory failure." B. "Oxygen will not be needed until or unless you are in the terminal stages of this disease." C. "Long-term home oxygen therapy should be used to prevent heart problems related to COPD." D. "You will not need oxygen until your oxygen saturation drops to 88% and you have symptoms of hypoxia."

D. "You will not need oxygen until your oxygen saturation drops to 88% and you have symptoms of hypoxia." Long-term oxygen therapy in the home will not be considered until the oxygen saturation is less than or equal to 88% and the patient has signs of tissue hypoxia, such as cor pulmonale, erythrocytosis, or impaired mental status. PaO2 less than 55 mm Hg will also allow home oxygen therapy to be considered.

Which medication should the nurse anticipate being used first in the emergency department for relief of severe respiratory distress related to asthma? A. Prednisone orally B. Tiotropium inhaler C. Fluticasone inhaler D. Albuterol nebulizer

D. Albuterol nebulizer

Tobacco smoke causes defects in multiple areas of the respiratory system. What is a long term effect of smoking? A. Bronchospasm and hoarseness B. Decreased mucus secretions and cough C. Increased function of the alveolar macrophages D. Increased risk of infection and hyperplasia of mucus glands

D. Increased risk of infection and hyperplasia of mucus glands

During an acute exacerbation of mild COPD, the patient is severely short of breath and the nurse identifies a nursing diagnosis of ineffective breathing pattern related to obstruction of airflow and anxiety. What is the best action by the nurse? A. Prepare and administer routine bronchodilators B. Perform chest physiotherapy to promote removal of secretions C. Administer O2 at 5 L/min until SOB is resolved D. Position the patient upright with the elbows resting on over the bed table

D. Position the patient upright with the elbows resting on over the bed table

The nurse is assisting a patient to learn self-administration of beclomethasone, two puffs inhaled every 6 hours. What should the nurse explain as the best way to prevent oral infection while taking this medication? A. Chew a hard candy before the first puff of medication B. Rinse the mouth with water before each puff of medication C. Ask for a breath mint following the second puff of medication D. Rinse the mouth with water following the second puff of medication

D. Rinse the mouth with water following the second puff of medication Because beclamethosone is a corticosteroid, the patient should rinse the mouth with water following the second puff of medication to reduce the risk of fungal overgrowth and oral infection.

The husband of a patient with severe COPD tells the nurse that he and his wife have not had any sexual activity since she was diagnosed with COPD because she becomes to short of breath. What is the nurses best response? A. You need to discuss your feelings and needs with your wife so she knows what you expect of her B. There are other ways to maintain intimacy besides sexual intercourse that will not make her SOB C. You should explore other ways to meet your sexual needs since your wife is no longer capable of sexual activity D. Would you like for me to talk to you and your wife about some modifications that can be made to maintain sexual activity?

D. Would you like for me to talk to you and your wife about some modifications that can be made to maintain sexual activity?


संबंधित स्टडी सेट्स

English Lesson Six - Rhetorical and Logical Fallacies

View Set

Health Chapter 5 "Infectious Disease"

View Set

Chapter 3 Legal and Ethical Aspects of Nursing

View Set

Review Questions Network+ Module 2

View Set